economy grand test sub sect1

You might also like

Download as pdf or txt
Download as pdf or txt
You are on page 1of 44

TSPSC - Group-2 – GRAND TEST (PAPER-II) (19-03-2023)

like the Gross Domestic Product (GDP),


Wholesale Price Index (WPI).
1) Consider the following statements about
● In India, IIP is a representative figure which
Index of Industrial Production (IIP):
measures the general level of Industrial
1. It measures the short-term changes in the
activity in the country. As IIP shows the status
volume of production of a basket of industrial
of industrial activity, one can find out if the
products during a given period.
industrial activity has increased, decreased or
2. It shows the volume of industrial activity.
remained the same. Being an abstract
3. The base year for IIP has been revised from
number, it does not show volume of activity
2004-05 to 2011-12.
and only shows the magnitude which
Which of the above statements are correct?
represents the status of production in the
(a) 1 and 2 Only
industrial sector for a given period of time as
(b) 1 and 3 Only
compared to a reference period of time.
(c) 2 and 3 Only
(d) 1, 2 and 3
Answer: B
2) What does the BIFR (Board for Industrial and
Explanation:
Financial Reconstruction) deal with?
Index of Industrial Production (IIP)
(a) Revival of sick industries.
● It measures the short-term changes in the
(b) Privatization of the public sector.
volume of production of a basket of industrial
(c) Approval to use of foreign capital in Indian
products during a given period with reference
industries.
to the chosen base year.
(d) Financing large scale private industries.
● Being an abstract number, it does not show
Answer: A
volume of activity and only shows the
Explanation:
magnitude which represents the status of
● The Board for Industrial and Financial
production in the industrial sector for a given
Reconstruction (BIFR) is a statutory authority
period of time as compared to a reference
established under The Sick Industrial
period of time.
Companies (Special Provisions) Act, 1985
● The Central Statistics Office (CSO), Ministry of
(SICA). The board was set up in January 1987
Statistics and Program Implementation has
.It works under the Department of Financial
revised the base year of the all-India Index of
Services of the Ministry of Finance.
Industrial Production (IIP) from 2004-05 to
● Its objective was to determine sickness of
2011-12 to not only reflect the changes in the
industrial companies and to assist in reviving
industrial sector but to also align it with the
those that may be viable and shutting down
base year of other macroeconomic indicators
the others.

KPIAS (1-B) CELL: 9133237733


● India is Infrastructure Finance Co. Ltd (IIFCL)
the main marketer of a credit enhancement
3) Disinvestment is: guarantee fund, announced in the 2016- 17
(a) Offloading of shares of private companies to budget. It will help to attract long- term
government investments particularly from global
(b) Offloading of government shares to private insurance, pension & sovereign wealth funds.
companies
(c) Increase in investment
(d) Closing down of business
Answer: B 5) The largest source of revenue for Britishers in India
Explanation: was from ______?
Disinvestment is a process in which offloading of (a) Textile and Jute Industry
government shares is done to private (b) Gems and Stone Industry
companies. A company or government (c) Temple Tax
organization will divest an asset or subsidiary (d) Land revenue
as a strategic move for the company planning Answer: D
to put the proceeds from the divestiture to Explanation:
better use that garners a higher return on The largest source of Revenue for Britishers in India
investment. was from Land revenue. Three major systems
of land revenue collection existed in India -
Zamindari, Ryotwari and Mahalwari.
4) Which of the following statements is/are correct
regarding Credit Enhancement Guarantee
Fund? 6) When was the first modern paper mill of the
1. It will enhance the credit rating of bonds issued by country set up?
infrastructure firms. (a) 1827
2. It will help to attract short-term investments (b) 1812
particularly from global insurance and (c) 1846
sovereign wealth funds. (d) 1854
3. The main promoter of the fund is India Answer: B
Infrastructure Finance Co. Ltd (IIFCL). Explanation:
Select the correct answer using the code given below: Paper and Paper Board is a forest-based industry. In
(a) 1 and 2 only 1812 the first modern paper mill of the
(b) 2 and 3 only country was set up at Serampore in West
(c) 1 and 3 only Bengal
(d) 1, 2 and 3
Answer: C
Explanation: 7) Which unit of the Hindustan Copper Ltd is the first
Credit Enhancement Guarantee Fund: copper smelting unit in India?
● It will improve the credit rating of bonds (a) Malanjkhand Copper Project (MP)
issued by infrastructure firms. (b) Khetri Copper Complex (Rajasthan)
● It is a long term initiative as large (c) Indian Copper Complex (Jharkhand)
infrastructure projects have long gestation (d) Taloja Copper Project (Maharashtra)
projects and give returns slowly. Answer: A
Explanation:

KPIAS (2-B) CELL: 9133237733


● Malanjkhand Copper Project was established Answer: D
in 1982. Initial project has been set up by Explanation:
Hindustan Copper Ltd to exploit the copper The correctly matched pairs are : Mundra SEZ —
ore through an open pit mine. Adani Group, CMC Ltd. — Tata Group
● Khetri Copper complex (1967): It is developed
& mainted by Hindustan copper ltd.
● Indian copper complex (1930): Process plant
to produce 19000 TPA of referred copper.
● Taloja copper project (1989): Plant to 10) In the latest budget, there is a major focus on
produce 60,000 TPA of continuous cast wire infrastructure development. In response to
rod. the pandemic ravaged economy, it is seen as:
(i) Demand side response
(ii) Supply side response
Select the correct answer using the code given below:
(a) (i) only
8) Kuwait’s biggest Mobile Telecommunication (b) (ii) only
Company known as Zain has been bought (c) Both (i) & (ii)
by— (d) Neither (i) nor (ii)
(a) Bharti Airtel Answer: C
(b) Reliance Communication Explanation:
(c) Vodafone When Govt. spends on infrastructure then in the
(d) Aircel short term it leads to employment generation
Answer: A to construct the infrastructure which puts
Explanation: money in the hands of people leading to
Bharti Airtel has acquired Zain Telecom’s African increased demand and it also increases the
assets in a $10.7 billion deal. The acquisition demand through the materials like steel,
of Zain assets will give the Indian mobile cement etc. In the medium to long term,
market leader a footprint in 15 African because of better infrastructure supply
countries. constraints are removed and the economy
becomes more efficient and cost of
production reduces.

9) Which of the following pairs are correctly


matched?
Enterprise - Industrial Group 11) Consider the following statements:
1. VSNL - Bharti Group (i) Retail and wholesale trade are now classified as
2. Mundra SEZ - Adani Group MSME enterprises
3. CMC Ltd. - Tata Group (ii) Retail and wholesale trade are treated as MSME
4. IPCL - Reliance Group enterprises only for availing priority sector
Select the correct answer using the codes given lending benefits
below: (iii) Street vendors can register on Udyam
(a) 1, 2 and 3 Registration Portal and avail the benefits of
(b) 3 and 4 priority sector lending
(c) 1, 2 and 4 Select the correct answer using the code given below:
(d) 2, 3 and 4 (a) (i) only

KPIAS (3-B) CELL: 9133237733


(b) (ii) & (iii) only ● The pipeline has been developed by NITI
(c) (ii) only Aayog, in consultation with infrastructure line
(d) (i) & (iii) only ministries.
Answer: B ● Modality of such unlocking is envisaged to be
Explanation: by way of structured contractual agreements
● MSMEs can register themselves on the (PPP agreements/concessions) as against
Udyam Registration Portal and are allocated a complete privatization and through
unique “Udyam Registration Number” which Infrastructure Investment Trusts (InvIT).
enables MSMEs to seek information and Assets would be handed back to the Govt. at
apply online about various services being the end of the contract/transaction period.
offered by all Ministries and Departments.
● Recently, the retail and wholesale trade was 13) Consider the following statements:
included as MSMEs and they are allowed to (i) Private Label products are sold under the name of
be registered on the Udyam Registration specific retailers
Portal. However, the benefits to retail and (ii) White Label products are sold by multiple sellers
wholesale trade MSMEs are to be restricted under their brand
to Priority Sector Lending only. Select the correct answer using the code given below:
● In this regard, now, street vendors can also (a) (i) only
register as retail traders on Udyam (b) (ii) only
Registration portal and avail the benefit of (c) Both (i) & (ii)
priority sector lending. (d) Neither (i) nor (ii)
Answer: C
Explanation:
● A private Label/brand is a good that is
12) Which of the following statements are true manufactured for and sold under the name of
regarding the ‘National Monetization a specific retailer. A private label product is
Pipeline’? developed for a specific seller and sold
(i) Plans for monetization of core assets of central exclusively under that seller's brand.
government and central PSUs ● A white label product is developed by a
(ii) The pipeline has been developed by Dept. of manufacturer and distributed to multiple
Investment and Public Asset Management in sellers. Each seller can then apply its name
consultation with Infrastructure ministries and packaging and resell the product under
(iii) Focuses on privatization of brownfield projects the seller's brand.
Select the correct answer using the code given below:
(a) (i) only
(b) (i) & (ii) only
(c) (ii) & (iii) only 14) Consider the following statements regarding
(d) (i) & (iii) only “National Pharmaceuticals Pricing Authority”
Answer: A (NPPA):
Explanation: (i) It is under ministry of health and family welfare
● NMP estimates aggregate monetization (ii) It fixes/revises the prices of controlled drugs
potential of Rs 6.0 lakh crores through core (iii) It implements the provisions of Drug Price Control
assets of the Central Government and Central Order (DPCO)
Public Sector Enterprises, over a four-year Select the correct answer using the code given below:
period, from FY 2021-22 to FY 2024-25. (a) (i) only

KPIAS (4-B) CELL: 9133237733


(b) (i) & (iii) only (DPIIT), Ministry of Commerce and Industries,
(c) (ii) & (iii) only Government of India.
(d) (i) & (iii) only ● Invest India is transforming the country’s
Answer: C investment climate by simplifying the
Explanation: business environment for investors. Its
● Drug Price Control Orders (DPCOs) are issued experts, specializing across different
by the Government, in exercise of the powers countries, Indian states and sectors,
conferred under section 3 of the Essential handhold investors through their investment
Commodities Act 1955 to ensure that the lifecycle from pre-investment to after-care.
medicines listed under National List of Invest India’s specialists provide multiple
Essential Medicines (NELM) are available at a forms of support such as market entry
reasonable price to the general public. strategies, deep dive industry analysis,
● The National List of Essential Medicines partner search and location assessment, and
(NELM), prepared by the Ministry of Health policy advocacy with decision makers.
and Family Welfare, is a list of medicines
considered essential and high priority for
India’s health needs. It is based on aspects
like prevalence of disease in the population,
safety and efficacy of the medicine, and 16) The term “Open access” in electricity means
current affordability. which of the following:
● NPPA is responsible for fixing and revising the (a) Consumers have access to the transmission and
prices of pharmaceutical products and distribution network to obtain electricity
availability of the medicines in the country as from the suppliers of their choice
well as the enforcement of DPCO. (b) Consumers can choose the distribution company
of their choice
(c) Any consumer of electricity can also generate
electricity from their own resource and can
15) Consider the following statements regarding feed on to the distribution network.
"Invest India": (d) Distribution companies can choose from which of
(i) It is a non-profit agency the power producers they want to purchase
(ii) It is an agency under Dept. of Economic Affairs power without government interference
Select the correct answer using the code given below: Answer: A
(a) (i) only Explanation:
(b) (ii) only Open access is the non-discriminatory use of
(c) Both (i) & (ii) transmission and distribution infrastructure
(d) Neither (i) nor (ii) of the licensees by consumers for procuring
Answer: A electricity from the source of their choice.
Explanation:
● "Invest India" is the National Investment 17) Which of the following statements are true
Promotion and Facilitation Agency of India regarding “Swiss challenge”?
and acts as the first point of reference for (a) It is a game
investors in India. Invest India is set up as a (b) It is a method of awarding projects by the
nonprofit venture under the Department for government
Promotion of Industry and Internal Trade (c) It is related to lottery business
(d) None of the above

KPIAS (5-B) CELL: 9133237733


Answer: B Governments and by employees of private
Explanation: institutions, self-employed professionals and
A Swiss challenge is a form of public procurement unorganized sectors.
operated in some jurisdictions, which
requires a public authority which has
received an unsolicited bid for a public
project, or for services to be provided to the 19) What do you understand about the term “Circular
government, to publish the bid and invite Economy” often seen in the news?
third parties to match or better it. (a) It refers to an industrial system that is restorative
or regenerative in nature
(b) It refers to the cyclical booms and recession in an
economy
(c) It refers to a “Closed Economy” that primarily
18) Arrange the following in the correct chronological relies on internal trade
order i.e. the one which was established as a (d) It refers to a system in which there is Cyclical
statutory body should come first in the order. correlation between savings and investment
(i) Pension Fund Regulatory Development Authority Answer: A
(PFRDA) Explanation:
(ii) Insurance Regulatory and Development Authority A circular economy is an industrial system that is
of India (IRDAI) restorative or regenerative by intention and
(iii) Securities and Exchange Board of India (SEBI) design. It replaces the end-of-life concept
Select the correct answer using the code given below: with restoration, shifts towards the use of
(a) (ii) - (i)- (iii) renewable energy, eliminates the use of toxic
(b) (iii) – (ii) – (i) chemicals, which impair reuse and return to
(c) (i) – (ii) – (iii) the biosphere, and aims for the elimination of
(d) (ii) – (iii) – (i) waste through the superior design of
Answer: B materials, products, systems and business
Explanation: models.
● The Securities and Exchange Board of India
was established on April 12, 1992 in
accordance with the provisions of the
Securities and Exchange Board of India Act,
1992. 20) Consider the following statements:
● As per the section 4 of IRDAI Act 1999, (i) Nuclear energy generation capacity share is less
Insurance Regulatory and Development than 10%
Authority of India (IRDAI) was constituted (in (ii) Renewable energy generation capacity share is
2000) by an act of parliament, Insurance more than 25%
Regulatory and Development Authority Act Select the correct answer using the code given below:
1999. (a) (i) only
● The Pension Fund Regulatory Development (b) (ii) only
Authority (PFRDA) Act was passed in (c) Both (i) & (ii)
September, 2013 and the same was notified (d) Neither (i) nor (ii)
on 1st February, 2014. PFRDA regulates the Answer: C
National Pension System, subscribed by Explanation:
employees of Govt. of India, State Generation Capacity from various sources:

KPIAS (6-B) CELL: 9133237733


● Coal - 55% (i) Indian Gas Exchange is a wholly owned subsidiary
● Gas - 6.5% of Indian Energy Exchange
● Nuclear - 1.8% (ii) Indian Gas Exchange is an online delivery-based
● Renewable (Hydro, Solar, Wind) - 36.7% gas trading platform
Select the correct answer using the code given below:
(a) (i) only
(b) (ii) only
(c) Both (i) & (ii)
21) Consider the following statements: (d) Neither (i) nor (ii)
(i) Renewable Energy Certificates (REC) are traded on Answer: C
Indian Energy Exchange and Power Exchange Explanation:
of India ● Indian Gas Exchange (IGX) is the first
(ii) Ministry of Power in consultation with ministry of nationwide online delivery-based gas trading
New and Renewable Energy sets target every platform launched on 15th June 2020.
year for Renewable Purchase Obligation ● IGX is a non-govt, unlisted company and is a
(RPO) for all States/UTs wholly owned subsidiary of Indian Energy
Select the correct answer using the code given below: Exchange (IEX).
(a) (i) only ● IGX platform for natural gas has opened a
(b) (ii) only new chapter in the energy history of India
(c) Both (i) & (ii) and will help the nation move towards free
(d) Neither (i) nor (ii) market pricing of natural gas.
Answer: C
Explanation: 23) Consider the following statements regarding
Once a Renewable Power Generator/provider has fed ‘National Investment and Infrastructure Fund’
the energy generated into the electricity grid, (NIIF):
he receives REC (which can then be sold on (i) It is meant for both greenfield and brownfield
the open market as an energy commodity). project
RECs are a market-based instrument which (ii) It will invest into infrastructure projects and
certifies that the bearer (holder of REC) owns infrastructure financing companies
one megawatt-hour (MWh) of electricity (iii) It will raise funds from both domestic and
generated from a renewable energy international sources
resource. RECs are proof that energy has Select the correct answer using the code given below:
been generated from renewable sources such (a) (ii) only
as solar or wind power etc. When someone (b) (ii)& (iii) only
purchases RECs, renewable energy is (c) (iii) only
generated on his/her behalf. RECs can go by (d) All of the above
many names, including Green tag, Tradable Answer: D
Renewable Certificates (TRCs), Renewable Explanation:
Electricity Certificates, or Renewable Energy Government established NIIF in 2015 with the aim to
Credits. attract investment from both domestic and
international sources for funding
commercially viable Greenfield, Brownfield
and stalled projects in the infrastructure
sector. NIIF has been formed as a trust and is
22) Consider the following statements: registered with SEBI under Category II of

KPIAS (7-B) CELL: 9133237733


Alternative Investment Fund (for tax benefit). of Communications governs the fund and
It is basically a quasi-sovereign wealth fund as related provisions.
the government holds only 49% ownership. ● The aim of USOF is to provide a balance
NIIF will get funds from: between the provision of Universal Service to
● Overseas sovereign/quasi-sovereign/ all uncovered areas, including the rural areas.
multilateral/bilateral investors through
equity. Cash rich central PSU, provident
funds, insurance funds can also invest in NIIF 25) Consider the following statements regarding the
over and above Govt. of India share. SFURTI scheme:
● Market borrowings (debt). (i) It is implemented by ministry of MSME
NIIF will invest in: (ii) It promotes cluster-based development
● Infrastructure projects through equity and Select the correct answer using the code given below:
debt both; and (a) (i) only
● Non-Banking Financial Companies (NBFCs) (b) (ii) only
and Financial Institutions (FIs) involved in (c) Both (i) & (ii)
infrastructure financing through equity (d) Neither (i) nor (ii)
Answer: C
Explanation:
24) Which of the following statements are true The Ministry of MSME is implementing a ‘Scheme of
regarding Universal Service Obligation Fund Fund for Regeneration of Traditional
(USOF)? Industries’ (SFURTI) Scheme under which
(i) This fund is under Dept. of Telecommunication financial support is being provided for setting
(ii) This fund is used to provide ICT services in rural up of traditional industries clusters viz. Khadi,
and remote areas Coir & Village industries clusters.
(iii) This fund is created out of the budgetary
resources of Govt. of India 26) Which of the following statements are true with
Select the correct answer using the code given below: reference to Special Economic Zones (SEZ)?
(a) (i) only (i) SEZ units may import/procure goods and services
(b) (i) & (ii) only from Domestic Tariff Area (DTA) without
(c) (iii) only payment of customs/import duty
(d) (ii) & (iii) only (ii) Customs duty is imposed on sale from SEZ to DTA
Answer: B (iii) SEZ units should be net foreign exchange earners
Explanation: Select the correct answer using the code given below:
● Universal Service Obligation Fund (USOF) is (a) (i) only
the pool of funds generated by 5% Universal (b) (i) & (iii) only
Service Levy that is charged upon all the (c) (iii) only
telecom fund operators on their Adjusted (d) All of the above
Gross Revenue (AGR). This fund is deposited Answer: D
in the Consolidated Fund of India and is Explanation:
dispatched on the approval of the Indian ● “Domestic Tariff Area” (DTA) means the
Parliament. whole of India (including the territorial
● The USOF comes under the Indian Telegraph waters and continental shelf) but does not
Act 1885. The act was amended in 2003 to include the areas of the Special Economic
give statutory status to the fund. The Zones (SEZs).
Department of Telecommunications, Ministry

KPIAS (8-B) CELL: 9133237733


● “Special Economic Zone” (SEZ) is a specifically ● Emergency Credit Line Guarantee (ECLG)
delineated duty-free enclave and shall be scheme is a loan facility for which 100%
deemed to be foreign territory for the guarantee would be provided by National
purposes of trade operations and duties and Credit Guarantee Trustee Company (NCGTC)
tariffs.SEZ units may be set up for to Banks/NBFCs/Financial Institutions for
manufacture of goods and rendering of lending to MSMEs.
services. Goods and services going into the ● It is extended in the form of additional
SEZ area from DTA shall be treated as exports working capital/term loan facility to
and goods coming from the SEZ area into DTA MSMEs/Pradhan Mantri Mudra Yojana
shall be treated as if these are being borrowers/Self employed/ Professions who
imported. have taken loans for business purposes.
● This facility is available to those who have
already borrowed but have not been able to
repay their outstanding (yet to be paid) loan.
● It is a pre-approved loan (you will be asked to
27) Which of the following agencies / ministry take loan and if you do not want you can opt
regulates tariffs for aeronautical services at out) and hence no processing charges and no
major airports? collateral will be required from the
(a) Airport Authority of India (AAI) borrowers.
(b) Airport Economic Regulatory Authority (AERA) ● This scheme is being widened to include
(c) Directorate General of Civil Aviation (DGCA) various other businesses and no need to mug
(d) Ministry of Civil Aviation up all the facts and figures. Just understand
Answer: B basic things
Explanation:
The Airports Economic Regulatory Authority (AERA) is
a statutory body constituted under the
Airports Economic Regulatory Authority of 29) Consider the following statements regarding the
India Act, 2008 with its head office at Delhi. philosophy of Atma Nirbhar Bharat:
(i) It promotes self sufficiency through import
substitution
28) Consider the following statements regarding the (ii) Producing goods where we have comparative
‘Emergency Credit Line Guarantee Scheme’ advantage and importing goods where we do
launched under the Atma Nirbhar Bharat not have that advantage
package: (iii) Liberalizing the economy and giving more space
(i) Its focus is on MSMEs to the private sector
(ii) Government agencies will provide credit (iv) Restricting foreign capital
guarantee Select the correct answer using the code given below:
(iii) It is a pre approved loan scheme (a) (i) & (iii) only
Select the correct answer using the code given below: (b) (ii) & (iii) only
(a) (i) & (ii) only (c) (ii), (iii) & (iv) only
(b) (ii) & (iii) only (d) (i) & (iv) only
(c) (i) & (iii) only Answer: B
(d) All of the above Explanation:
Answer: D Atmanirbhar Bharat should not merely be a classical
Explanation: import substitution policy that focusses just

KPIAS (9-B) CELL: 9133237733


on increasing domestic production; the d. 1, 2 and 3
emphasis should be on producing globally Ans: B
competitive products domestically and it • It is conducted every 10 years since 1872.
wont restrict foreign capital. • Census 2011 was the 15th census of India &
7th census after Independence.
• The motto of census 2011 was “Our Census,
Our future”.

30) In Aatma Nirbhar Bharat, the economy has to


move from ‘command and control’ to ‘plug 32. Consider the following statements about the
and play’. In this statement, ‘plug and play’ phases in the 15th Indian census:
refers to: 1. The 15th Indian Census was conducted in two
(a) Government will be playing a leading role in the phases, house listing and population
economic growth process enumeration.
(b) Government will provide policy and other support 2. Information for National Population Register
to pull-in private sector (c) Government will (NPR) was also collected in the first phase
provide red carpet for foreign investors 3. 2011 marks the first time biometric
(d) Government will provide digital infrastructure for information was collected
the economy Select the correct codes:
Answer: B a. 1 and 2 only
Explanation: b. 2 and 3only
Addressing the 95th annual plenary session of the c. 1 and 3 only
Indian Chamber of Commerce in Kolkata on d. 1, 2 and 3
11th June 2020, our Prime Minister said that Ans: D
the time has come to take the Indian • The 15th Indian Census was conducted in two
Economy from "Command and Control to phases, house listing and population
Plug and Play". enumeration.
• House listing phase began on 1 April 2010
and involved collection of information about
----------------------------------------------------------------------- all buildings. Information for National
---------------------------- Population Register (NPR) was also collected
----------------------------------------------------------------------- in the first phase, which will be used to issue
------------------------------- a 12-digit unique identification number to all
registered Indian residents by Unique
31. Consider the following statements about the Identification Authority of India (UIDAI).
Census of India: • The second population enumeration phase
1. It is conducted every 10 years since 1882. was conducted between 9 and 28 February
2. Census 2011 was the 15th census of India & 2011. Census has been conducted in India
7th census after Independence. since 1872 and 2011 marks the first time
3. The motto of census 2011 was “Our Census, biometric information was collected.
Our future”.
Select the correct codes: 33. Consider the following information with respect
a. 1 and 2 only to the statistics in the 15th Indian census:
b. 2 and 3only 1. No of states and UTs: 28 states and 9 union
c. 1 and 3 only territories

KPIAS (10-B) CELL: 9133237733


2. No of districts: 640 9. Any one above age 10 who can read and
3. No. of Sub-Districts: 5924 write in any language with an ability to
4. No of towns: 7,935 understand was considered a literate.
5. No of villages: 6,40,930 10. In censuses before 1991, children below the
Select the correct codes: age 7 were treated as illiterates.
a. 1,2,3 and 4 only 11. The literacy rate taking the entire population
b. 2,3,4 and 5only into account is termed as crude literacy rate
c. 1,2,3 and 5 only and taking the population from age 10 and
d. 1, 2,3,4 and 5 above into account is termed as effective
literacy rate
Ans: D Select the correct codes:
a. 1,2 only
1. No of states and UTs 28 states and 9 union b. 2,3 only
territories c. 1, 2 and 3
2. No of districts 640 d. Neither 1 nor 2 nor 3
3. No. of Sub-Districts 5924 Ans: D
4. No of towns 7,935 • Any one above age 7 who can read and write
5. No of villages 6,40,930 in any language with an ability to understand
was considered a literate.
34. List the following states in decreasing order of the • In censuses before 1991, children below the
total speakers of a particular language as a age 5 were treated as illiterates.
percentage of total population • The literacy rate taking the entire population
4. Hindi into account is termed as crude literacy rate
5. Bengali and taking the population from age 7 and
6. English above into account is termed as effective
7. Telugu literacy rate
8. Marathi
Select the correct codes: 36. List the following religious groups in decreasing
a. 12354 order of its population as a percentage of
b. 14523 total population
c. 15432 12. Christianity
d. 13254 13. Buddhism
Ans: D 14. Sikhism
Language Total speakers as a percentage of total 15. Jainism
population Select the correct codes:
• Hindi 57.10 a. 1234
• English 10.60 b. 1423
• Bengali 8.90 c. 1432
• Marathi 8.20 d. 1324
• Telugu 7.80 Ans: D
• Christianity 2.30%
• Sikhism 1.72%
35. Which of the following statements about the • Buddhism 0.70%
definition of literacy rate is correct: • Jainism 0.37%

KPIAS (11-B) CELL: 9133237733


37. Consider the following statements about the 39. consider the following important parameters in
populated districts in India: the 15th Indian census:
1. Thane district of Maharashtra is the most 16. Population Density- 482
populated district of India. 17. Sex ratio- 950
2. Dibang Valley has the least density of 1 18. Literacy Rate Overall - 74%
person per sq. km Select the correct codes:
Select the correct codes: a. 1 only
a. 1 only b. 2 only
b. 2 only c. 3 only
c. Both 1 and 2 d. 1, 2 and 3
d. Neither 1 nor 2 Ans: C
Ans: C • Population Density -382
1. Thane district of Maharashtra is the most • Population-1210.19 million
populated district of India. • Sex ratio- 940
2. Dibang Valley has the least density of 1 • Literacy Rate Overall - 74%
person per sq. km

38) In which year the government of India announced 40) Which of the following statements are correct
its first population policy? with reference to the National Population
A. 1976 Policy of 2000?
B. 1990 A. The medium-term objective of the NPP 2000
C. 2000 was to reduce the Total Fertility Rate (TFR) to
D. 2012 replacement levels by 2010. The TFR was to
Answer: A have 2.1 children per woman.
Explanation: B. The long-term objective is “ to achieve a
In 1976, GOI announced the first National Population stable population by 2045, at a level
Policy. Some of the measures to check the consistent with the requirements of
population growth as part of this policy sustainable economic growth, social
include: development, and environmental
● Increased the minimum legal marriageable protection.”
age for boys and girls to 21 and 18 C. Both A and B
respectively. D. Neither A nor B
● Providing monetary incentives for employing Answer: C
birth control. Explanation:
● Improving women’s literacy levels through The National Population Policy (NPP), 2000 is the
formal and informal channels. central government’s second population
● Population was made a criteria in deciding policy. The NPP states its immediate
the quantum of central assistance to states. objective as addressing the unmet needs for
● Using the different forms of media to contraception, healthcare infrastructure, and
popularise family welfare programmes. health personnel, and providing integrated
● Introducing population education into the service delivery for basic reproductive and
formal education system. child healthcare.
● The medium-term objective of the NPP 2000
was to reduce the Total Fertility Rate (TFR) to

KPIAS (12-B) CELL: 9133237733


replacement levels by 2010. The TFR was to 43. Consider the following statements about the
have 2.1 children per woman. tribal population in India as per the 15th
● The long-term objective is “ to achieve a Indian census:
stable population by 2045, at a level a. There are nearly 550 tribes i.e. 8.2% of the
consistent with the requirements of total Population of India i.e. 10 million
sustainable economic growth, social population
development, and environmental b. Jharkhand has highest tribal population in
protection.” India.
c. Haryana has zero tribal population
Select the correct codes:
41) The National Population Policy of India has set the a. 1 and 2 only
following goals except : b. 2 and 3 only
A. To bring down Total Fertility Rate (TFR) to c. 1 and 3 only
replacement levels by 2015 d. 1, 2 and 3
B. To reduce the Infant Mortality Rate to 30 per Ans: C
l000 live births Tribe
C. To reduce the Maternal Mortality Rate to 100 • 550 tribes i.e. 8.2% of the total Population of
per 100000 live births India i.e. 10 million population
D. 100 percent registration of births, deaths, Population -
marriages and pregnancies • MP (1.5 million)
Answer : A. Density -
Explanation: • Mizoram
The goal of The National Population Policy of India is Population -
to bring down Total Fertility Rate (TFR) to • Punjab (zero)
replacement levels by 2010. • Haryana (zero)
Density -
• Punjab (zero)
42. Consider the following important statistics about • Haryana (zero)
literacy rate in the 15th Indian census:
1. The difference between male and Female
literacy rate is only 10% 44. Consider the following statements about the
2. Kerala has highest literacy rate while Bihar settlement development in India as per the
has the least 15th Indian census:
Select the correct codes: d. Rajasthan is the largest state by area
a. 1 only e. West Bengal has the highest slum population
b. 2 only in India
c. Both 1 and 2 f. Goa is the highest urbanized state in India.
d. Neither 1 nor 2
Ans: b Select the correct codes:
• Male - 82.14% Female - 65.46 a. 1 and 2 only
• Kerala (93.9%) Bihar (63.80%) b. 2 and 3 only
c. 1 and 3 only
d. 1, 2 and 3
Ans: C

KPIAS (13-B) CELL: 9133237733


• State (Area) Rajasthan has highest area and Bihar has the highest percentage of population below
Goa has the least area. poverty line and Goa has the lowest
• Slums: Maharashtra has the highest slum population below poverty line.
population and Arunachal Pradesh has the
least. 47) A regressive population age structure is likely to
• Urbanisation: Goa has the highest be encountered in a country:
urbanization and Himachal Pradesh has the 1. which has completed the demographic
least transition
2. where the young and old population are
balanced
45. Consider the following statements about the Child 3. which has young population
sex ratio in India as per the 15th Indian 4. which is in the early expanding stage of
census: demographic transition
g. Child sex ratio is the number of girl child per Answer: A
10000 boys in the age group of 0-6 Explanation:
h. As per the 2011 census the CSR is 914 A regressive population age structure is characterized
Select the correct codes: by a relatively high proportion of elderly
a. 1 only people and a low proportion of young
b. 2 only people. This can be caused by low birth rates,
c. Both 1 and 2 high death rates, and increased life
d. Neither 1 nor 2 expectancy.
Ans: B
• Child sex ratio (female per 1000 male in the
age group of 0-6) 48) Which of the following statements are correct?
• The CSR is 914. It is highest in Mizoram and 1. In 1956, a Central Family Planning Board was
lowest in Haryana set up in India to focus on sterilization
2. China became one of the first developing
countries to come up with a state-sponsored
46. Consider the following statements about the family planning programme in the 1950s.
poverty lines in India as per the 15th census: Options:
i. Percentage of Population Below Poverty Line A. Only 1
(Tendulkar Methodology) was 21% B. Only 2
j. Bihar has the highest percentage of C. Both 1 and 2
population below poverty line and Goa has D. Neither 1 nor 2
the lowest population below poverty line. Answer: A
Select the correct codes: Explanation:
a. 1 only ● India became one of the first developing
b. 2 only countries to come up with a state-sponsored
c. Both 1 and 2 family planning programme in the 1950s.
d. Neither 1 nor 2 ● A population policy committee was
Ans: B established in 1952. However, the policies
Percentage of Population Below Poverty Line framed in the early fifties were largely
(Tendulkar Methodology) was 29.8% arbitrary and so not successful.
● In 1956, a Central Family Planning Board was
set up and its focus was on sterilisation.

KPIAS (14-B) CELL: 9133237733


50) Which of the following index summarizes the
population dynamics all together?
49) Which of the following committees are related to A. Fertility rate
the framing of Population policies in India? B. Gini coefficient
1. Radha Kamal Mukherjee Committee C. Demographic Index
2. Bhore Committee D. Mortality rate
3. Dr. MS Swaminathan Committee Answer: C
Options: Explanation:
A. Only 1 Demographic Index can summarize population
B. Only 1, 2 dynamics and characteristics in one number.
C. Only 1, 3
D. All of the above
Answer: D
Explanation: 51) Which of the following factors are responsible for
● Radha Kamal Mukherjee Committee (1940): sprase population?
In 1940, the Indian National Congress (A) Flat plains and abundant rainfall
appointed a Committee headed by a social (B) Rugged terrain and unfavorable climate
scientist Radha Kamal Mukherjee to suggest (C) Fertile soil and abundant rainfall
solutions to arrest the population which has (D) Rugged terrain and favorable climate
started increasing rapidly after 1921. The Answer: B
committee recommended self-control, Explanation:
generating awareness of cheap and safe birth Uneven terrain and unfavorable climatic conditions
control measures, discouraging polygamy, are prime reasons for the sparse population.
among others, as measures to bring down the Many states are situated in hilly, arid, or
rate of population growth. semi-arid regions. Such terrains are difficult
● Bhore Committee: The Health Survey and to access or occupy.
Development committee under Sir Joseph
Bhore recommended ‘deliberate limitation of
family’ as a measure to control the 52) What year is considered a great demographic
population growth. This committee was set divide in India?
up in 1943 and submitted its report in 1946. (A) 1911
● A Committee on Population was appointed in (B) 1921
1991 which submitted its report in 1993 in (C) 1931
which it recommended the formu-lation of a (D) 1751
National Population Policy to take a ‘a long- Answer: B
term holistic view of development, Explanation:
population growth, and environ-mental The year 1921 is regarded as the defining year for
protection’ and to ‘suggest policies and Indian demography. It is also known as the
guidelines [for] formulation of programmes’ year of the Great Divide, as population
and ‘a monitoring mechanism with short- growth was scarce before that time. After
medium- and long-term perspectives and 1921, India's population growth remained
goals’. Accordingly, an Expert Group headed consistent.
by Dr. MS Swaminathan was set up to create
the draft national population policy.

KPIAS (15-B) CELL: 9133237733


53) Which of the following initiatives have been taken ● Scheme for provision of Pregnancy Testing
under the National Family Planning Kits in the drug kits of ASHA for use in
Programme? communities.
1. Mission Parivar Vikas ● Family Planning Logistics Management
2. Expanded Contraceptive Choices Information System (FP-LMIS): dedicated
3. Post-partum Intrauterine contraceptive software has been launched to ensure
device smooth forecasting, procurement and
Options: distribution of family planning commodities
A. Only 1 across all the levels of health facilities.
B. Only 1, 2
C. Only 1, 3
D. All of the above 54. Match the following?
Answer: D 1. Auguste Tano Kouame a. Executive
Explanation: secretary of climate change
Various initiatives have been taken under the 2. Subramaniam b. world Bank
National Family Planning Programme country director
providing broad range of services mentioned 3. Simon stiell c.
as given below: Executive director at IMF
● Mission Parivar Vikas has been introduced for A. 1-b, 2-c, 3-a
substantially increasing access to B. 1-b, 2-a, 3-c
contraceptives and family planning services in C. 1-a, 2-b. 3-c
146 high fertility districts in 7 high focus D. 1-c, 2-b, 3-a
states. Answer: A
● Expanded Contraceptive Choices: The current Explanation: UN Secretary-General Antonio Guterres
contraceptive basket comprising of condoms, has appointed Simon Stiell as the new
Combined oral contraceptive pills, Emergency Executive Secretary of the United Nations
contraceptive pills, Intrauterine contraceptive Climate Change Secretariat based in Bonn,
uterine device (IUCD) and Sterilization has Germany. The World Bank has appointed
been expanded with inclusion of new Cote d’Ivoire national, Auguste Tano Kouame,
contraceptives namely Injectable as the Country Director for India. The term of
contraceptive (Antara programme) and Auguste is effective from August 01, 2022, for
Centchroman (Chhaya). a period of five-years. Former Chief Economic
● Compensation scheme for sterilization Adviser of India K. Subramaniam was recently
acceptors which provides compensation for appointed as the Executive Director of India
loss of wages to the beneficiary and also to at the International Monetary Fund (IMF). He
the service provider team for conducting replaced Surjit S Bhalla
sterilisation.
● Post-partum Intrauterine contraceptive
device (PPIUCD) services are provided post- 55. Royal Enfield has tied up with UNESCO, reason is?
delivery. A. For advertising
● Scheme for Home Delivery of contraceptives B. Promoting awareness about diseases
by ASHAs at the doorstep of beneficiaries has C. Safeguard intangible cultural heritage
been taken up. D. None
Answer: C

KPIAS (16-B) CELL: 9133237733


Explanation: Royal Enfield has partnered with 3. As of 2022, India and Israel have completed
UNESCO (the United Nations Educational, 30 years of diplomatic relations
Scientific and Cultural Organisation) to A. Only 2 and 3
promote and safeguard the ‘Intangible B. Only 1 and 3
Cultural Heritage of India, beginning with the C. Only 3
Himalayas. The programme is curated as an D. Only 1
experiential and creative showcase of the Answer: B
Intangible Cultural Heritage (ICH) practices in Explanation: Recently Egypt released a Postage Stamp
the Western Himalayas and the North celebrating the 75th Anniversary of
Eastern region diplomatic ties with India. Diplomatic
relations between Egypt and India were
established with Egypt’s recognition of the
56. Choose the correct answer? independence of India on 18 August 1947,
1. India and Nepal have entered an agreement just three days after India’s independence.
on water sharing of Kushiyara river The Diplomatic relations between India and
2. India and Bangladesh have entered an Israel were established in 1992
agreement on water sharing of Kushiyara
river 58. Mexican President Obrador proposed Commission
3. Hindustan Aeronautics Limited has opened its of global Notables, who among the following
first international marketing office in is not part of it?
Indonesia A. Narendra Modi
4. Hindustan Aeronautics Limited has opened its B. Antonio Guterres
first international marketing office in C. Joe Biden
Malaysia D. Pope Francis
A. Only 1 and 3 are true
B. Only 2 and 3 are true Answer: C
C. Only 1 and 4 are true Explanation: Mexican President Obrador has
D. Only 2 and 4 are true proposed the setting up of a commission
Answer: D called ‘Commission of Global Notables’
Explanation: The Hindustan Aeronautics Limited comprising Prime Minister Narendra Modi.
(HAL) inked a Memorandum of Apart from Mr. Modi, the proposed
Understanding (MoU) to establish its first “commission of global notables” includes
international marketing and sales office in Pope Francis and the UN Secretary-General
Kuala Lumpu. Recently, India and Bangladesh Antonio Guterres.
have finalised the text of Memorandum of
Understanding (MoU) on interim water 59. Choose the correct answer?
sharing of Kushiyara river 1. The statue of freedom fighter and agricultural
scientist Panduranga Khankoje was unveiled
in Mexico
57. Choose the correct answer? 2. The statue of freedom fighter and agricultural
1. India and Egypt have established diplomatic scientist Panduranga Khankoje was unveiled
relations in 1947 in Canada
2. India and Egypt have established diplomatic 3. The Bust of Mahatma Gandhi was unveiled in
relations in 1950 Ecuador

KPIAS (17-B) CELL: 9133237733


4. The bust of Mahatma Gandhi was unveiled in 2. It deals in lending and remittance services only.
Paraguay 3. Its funds are mainly invested in government papers
A. Only 1 and 4 are true and bank deposits.
B. Only 2 and 3 are true 4. It cannot accept demand deposits and credit cards.
C. Only 1 and 3 are true Which of the above statements are correct?
D. Only 2 and 4 are true (a) 2, 3 and 4 only
Answer: A (b) 2 and 4 only
Explanation: External affairs minister S Jaishankar (c) 1 and 3 only
unveiled a bust of Mahatma Gandhi in (d) 1, 2 and 3
Paraguay and visited the historic Casa de la Answer: C
Independencia, from where the South Explanation:
American country’s independence movement ● A payments bank is like any other bank, but
started more than two centuries ago. The operating on a smaller scale without involving
statue of freedom fighter and agricultural any credit risk. It can carry out most banking
scientist Pandurang Khankhoje will soon be operations but can’t advance loans or credits.
unveiled in Mexico ● A Payment Bank will need to invest 75% of its
funds in government securities or treasury
60 .Choose the correct answer? bills.
1. World’s highest Railway Bridge was ● The main objective of Payment Banks is to
constructed on Chenab River accelerate financial inclusion.
2. India’s longest train is Super Vasuki ● Payments banks will mainly deal in transfer
A. Only 1 is true and remittance services and accept deposits
B. Only 2 is true of up to Rs 1 lakh. Payment Banks will not
C. None is true lend to customers.
D. Both are true ● Payment Banks will have to deploy their
Answer: D funds in government papers and bank
Explanation: The golden joint of the world’s highest deposits.
railway bridge over River Chenab was ● They can accept demand deposits, issue
inaugurated. For the first time ever since ATM/debit cards but not credit cards.
independence, Srinagar will be linked to the
rest of India after the overarch deck on the
world’s highest single-arch railway bridge 62) Which of the following sectors represent the
over River Chenab. The bridge will be 35 categories of Priority Sector Lending?
meters higher than the Eiffel Tower. The 1. Mining
Railways conducted a test run of its longest 2. Social Infrastructure
freight train, Super Vasuki, with 295 loaded 3. Export Credit
wagons carrying over 27,000 tonnes of coal. 4. Education
The 3.5-km-long freight train covered the Select the correct answer using the code given below:
distance of about 267 km between Korba in (a) 1 and 3 only
Chhattisgarh and Rajnandgaon in Nagpur (b) 1 and 2 only
(c) 2, 3 and 4 only
(d) 1, 2, 3 and 4
61) Consider the following statements with reference Answer: C
to Payment Bank: Explanation:
1. Its objective is to accelerate financial inclusion.

KPIAS (18-B) CELL: 9133237733


As per the RBI circular, there are eight broad ● Simplified Branch Authorization Policy.
categories of the Priority Sector Lending ● Compulsory Requirement of Opening
which are:- Agriculture, Micro, Small and Branches in Unbanked Villages.
Medium Enterprises, Export Credit, ● Setting of Financial Literacy Centers.
Education, Housing, Social Infrastructure, Financial inclusion is the means to achieve
Renewable Energy and Others. sustainable development goals.

63) Consider the following statements regarding 64) Promoting Digital Payments has been mandated
Financial Inclusion? to which of the following?
1. It aims at better access to useful and affordable (a) Ministry of Electronics and Information
financial products for individuals and Technology (MeitY)
businesses (b) NITI Aayog
2. Opening of transaction account is the first step (c) PMO
towards financial inclusion (d) Ministry of Finance
3. Financial inclusion helps in achieving sustainable Answer: A
development goals. Explanation:
Which of the above statements is/are correct? ● The responsibility to promote digital
(a) 1 only transactions in India has been shifted from
(b) 1 and 3 only NITI Aayog to the Ministry of Electronics and
(c) 2 only Information Technology (MeitY). By shifting
(d) 1, 2 and 3 the responsibility to promote digital
Answer: D transactions to MEITY, the government is
Explanation: trying to utilize the core competence of
● Financial inclusion means that individuals and specific ministries.
businesses have access to useful and ● NITI Aayog would have a role to monitor and
affordable financial products and services recommend ways to improve government
that meet their needs – transactions, schemes and growth of IT- enabled services.
payments, savings, credit and insurance –
delivered in a responsible and sustainable 65) Which of the following is/are the objective(s) of
way. financial literacy camps?
● Being able to have access to a transaction 1. It aims at imparting knowledge to enable financial
account is a first step toward broader planning.
financial inclusion since a transaction account 2. It aims at inculcating saving habits.
allows people to store money, and send and 3. It aims at improving the understanding of financial
receive payments. A transaction account products leading to effective use of financial
serves as a gateway to other financial services by the common man.
services Select the correct answer using the code given below:
● Moving from access to account to account (a) 3 Only
usage is the next step (b) 1 and 3 Only
Government Initiatives for financial inclusion are: (c) 2 and 3 Only
● Setting up of Business Correspondents (d) 1, 2 and 3
model. Answer: D
● Setting of Basic Saving Bank Deposit (BSBD) Explanation:
accounts. Financial Literacy camps:

KPIAS (19-B) CELL: 9133237733


● It aims at imparting knowledge to enable ● Fiscal deficit is defined as excess of total
financial planning, inculcate saving habits and budget expenditure over total budget
improve the understanding of financial receipts excluding borrowings during a fiscal
products leading to effective use of financial year. In simple words, it is the amount of
services by the common man borrowing the government has to resort to
● The objective of conducting financial literacy meet its expenses. A large deficit means a
camps is to facilitate financial inclusion large amount of borrowing. Fiscal deficit is a
through provision of two essentials i.e. measure of how much the government needs
literacy and easy access. to borrow from the market to meet its
● It aims at imparting knowledge to enable expenditure when its resources are
financial planning, inculcate saving habits and inadequate.
improve the understanding of financial ● Fiscal deficit = Total expenditure – Total
products leading to effective use of financial receipts excluding borrowings
services by the common man. ● If we add borrowing in total receipts, the
● Financial literacy should help them plan fiscal deficit is zero. Clearly, the fiscal deficit
ahead of time for their life cycle needs and gives the borrowing requirements of the
deal with unexpected emergencies without government.
resorting to debt. They should be able to ● High fiscal deficit results in high government
proactively manage money and avoid debt borrowing which leads to lowering the
traps. In order to ensure that the knowledge amount of available money to be borrowed
provided through awareness results in by private investors, increasing the interest
inculcating banking habits, literacy inputs rates and lowering the private investments.
need to be synchronized with access to ● The Central bank no longer finances the
financial services so as to enable the common deficits of the Government. It is a facilitator
man to use the information effectively to gain of borrowings from the market.
control over financial matters. It should also
result in enhancement of their economic
security aided by use of banking services. 67) Consider the following statements regarding the
Public Financial Management System (PFMS):
66) Which of the following statements is/are correct 1. It is a web-based online software application
related to Fiscal Deficit? developed and implemented by the Office of
1. It shows the excess of Total Expenditure over Total Controller General of Accounts.
Receipts excluding Borrowings. 2. The biggest strength of PFMS is its integration with
2. High Fiscal Deficit crowds out private investment in the Core banking system in the Country.
normal times. Which of the above statements is/are correct?
3. The Fiscal Deficit is financed through borrowing (a) 1 only
from the Central Bank. (b) 2 only
Select the correct answer using the code given below: (c) Both 1 and 2
(a) 1 and 2 Only (d) Neither 1 nor 2
(b) 2 Only Answer: C
(c) 2 and 3 Only Explanation:
(d) 1, 2 and 3 The Public Financial Management System (PFMS):
Answer: A ● The Public Financial Management System
Explanation: (PFMS), earlier known as Central Plan
Schemes Monitoring System (CPSMS), is a

KPIAS (20-B) CELL: 9133237733


web- based online software application
developed and implemented by the Office of 69) According to the Union Budget 2023-24, consider
Controller General of Accounts (CGA). the following statements. The Budget adopts
● The biggest strength of PFMS is its integration the following seven priorities:
with the Core banking system in the Country. 1. Inclusive Development
As a result, PFMS has the unique capability to 2. Reaching the Last Mile
push online payments to almost every 3. Infrastructure and Investment
beneficiary/vendor. At present, PFMS 4. Unleashing the Potential
interface is having interface in addition to the Which among the following is/are not one among
Core Banking System (CBS) of all Public Sector them?
Banks, Regional Rural Banks, major private (a) 1 and 2
sector banks, Reserve Bank of India, India (b) 2 and 3
post and Cooperative Banks. (c) 1 and 4
(d) None of the above
Answer: d
68) Consider the following statements regarding the Explanation:
Public Debt Management Agency (PDMA) The Budget adopts the following seven priorities.
1. It manages only internal liabilities of the Central They complement each other and act as the
and state governments. ‘Saptarishi’ guiding us through the Amrit Kaal.
2. It is the Investment Banker or Merchant Banker to 1. Inclusive Development
the Government. 2. Reaching the Last Mile
Which of the above statements is/are incorrect? 3. Infrastructure and Investment
(a) 1 only 4. Unleashing the Potential
(b) 2 only 5. Green Growth
(c) Both 1 & 2 6. Youth Power
(d) Neither 1 nor 2 7. Financial Sector
Answer: A 70) According to the Union Budget 2023-24, consider
Explanation: the following statements.
● Public Debt Management Agency (PDMA) is a 1. Agriculture Accelerator Fund will be set-up to
specialised independent agency that encourage agri startups by young
manages the internal and external liabilities entrepreneurs in urban areas.
of the Central Government in a holistic 2. The Fund will aim at bringing innovative and
manner and advises on such matters in affordable solutions for challenges faced by
return for a fee. farmers.
● In other words, PDMA is the Investment Which of the above statements is/are correct?
Banker or Merchant Banker to the (a) 1 only
Government. PDMA manages the issue, (b) 2 only
reissue and trading of Government securities, (c) Both 1 and 2
manages and advises the Central (d) Neither 1 nor 2
Government on its contingent liabilities and Answer: (b)
undertakes cash management for the central Explanation:
government including issuing and redeeming An Agriculture Accelerator Fund will be set-up to
of short term securities and advising on its encourage agristartups by young
cash management. entrepreneurs in rural areas. The Fund will

KPIAS (21-B) CELL: 9133237733


aim at bringing innovative and affordable (d) 1, 2 and 3
solutions for challenges faced by farmers. Answer: c
Explanation:
● According to Article 112 of the Indian
71) According to the Union Budget 2023-24, consider Constitution, the Union Budget of a year is
the following statements. referred to as the Annual Financial Statement
1. One hundred and fifty-seven new nursing colleges (AFS).
will be established in co-location with the ● It is a statement of the estimated receipts
existing 157 medical colleges established and expenditure of the Government in a
since 2014 Financial Year (which begins on 1st April of
2. A Mission to eliminate Sickle Cell Anaemia by 2047 the current year and ends on 31st March of
will be launched. the following year).
Which of the above statements is/are correct?
(a) 1 only 73) Consider following statements regarding
(b) 2 only Contingency Fund of India
(c) Both 1 and 2 1. Currently, the Parliament has authorized a corpus
(d) Neither 1 nor 2 of ₹30000 crore.
Answer: (c) 2. It is to look for unforeseen expenditures.
Explanation: 3. It is placed at the disposal of Cabinet ministers.
● Nursing Colleges: One hundred and fifty- Which of the above statements are correct
seven new nursing colleges will be (a) 1 and 2
established in co-location with the existing (b) 1 and 3
157 medical colleges established since 2014. (c) 2 and 3
● Sickle Cell Anaemia Elimination Mission: A (d) 1, 2 and 3
Mission to eliminate Sickle Cell Anaemia by Answer: (a)
2047 will be launched. It will entail awareness Explanation:
creation, universal screening of 7 crore Contingency Fund of India:
people in the age group of 0-40 years in ● Provision for this fund is made in Article
affected tribal areas, and counselling through 267(1) of the Constitution of India.
collaborative efforts of central ministries and ● Its corpus is ₹30000 crore. It is in the nature
state governments. of an imprest (money maintained for a
specific purpose).
● The Secretary of, Finance Ministry holds this
72) Consider the following statements about the fund on behalf of the President of India.
Annual Financial Statement of 2023-24. ● This fund is used to meet unexpected or
1. This is provided in the Constitution under article unforeseen expenditure.
112. ● Each state can have its own contingency fund
2. It shows the actuals for the fiscal year 2022-23. established under Article 267(2)
3. Government accounts are kept under the
Consolidated Fund of India and the
Contingency Fund of India. 74) Which of the following statements about the
Which of the above statements are not correct? Indian budget is incorrect?
(a) 1 and 3 (a) Non-tax receipts of the government mainly consist
(b) 1 and 2 of interest and dividends on investment in
(c) 2 and 3

KPIAS (22-B) CELL: 9133237733


addition to fees and other receipts for ● The engineer- Statesman, Shri Visvesaraya,
services rendered by the government. was the first to advocate the idea of planning
(b) Interest payments on debt is part of the revenue for India.
expenditure. ● In his famous book, “The Planned Economy
(c) All grants to the state governments and union for India”, published in 1934, he proposed a
territories except for use of creation of ten-year’ plan with the aim of doubling the
capital assets. income of the country.
(d) None of the above ● His main emphasis was on industrialisation so
Answer: (c) as to reduce the population dependent on
Explanation: agriculture and to increase the population
All grants to the states or union territories are treated employed in industry.
as revenue expenditure even though a
portion of these may be used for creation of
capital assets. 77) With reference to Hindu Growth rate, consider
the following statements:
75) Which of the following statements is incorrect 1. It refers to the low annual growth rate of the
about the capital receipts? planned economy of India before the
(a) Loans from foreign governments and bodies liberalisations of 1991.
constitute capital receipts. 2. The term was coined by Indian economist Raj
(b) Disinvestment proceeds and dividend distribution Krishna.
are part of capital receipts. Which of the above statement is / are incorrect?
(c) Recoveries of loans from state governments are (a) 1 only
constituents of capital receipts. (b) 2 only
(d) Market borrowings from the public and (c) Both 1 and 2
borrowings through the sale of Treasury Bills (d) Neither 1 nor 2
(short- term borrowings) are part of capital Answer: D
receipts. Explanation:
Answer: (b) ● The Hindu rate of growth is a term referring
Explanation: to the low annual growth rate of the planned
Dividend distribution is part of revenue receipts economy of India before the liberalisations of
1991, which stagnated around 3.5% from
76) Consider the following statements: 1950s to 1980s, while per capita income
1. The first to advocate the idea of planning for India growth averaged 1.3%. The term contrasts
was Visvesaraya. with South Korea’s Miracle on the Han River
2. Visvesaraya proposed a 20 years plan with the aim and the Taiwan Miracle. While these Asian
of doubling the income of the country. Tigers had similar income level as India in the
3. ‘The Planned Economy For India’ was written by RC 1950s, exponential economic growth since
Dutt. then has transformed them into developed
Which of the above statement is/are correct? countries today.
(a) 1 only ● The term was coined by Indian economist Raj
(b) 2 and 3 only Krishna. It suggests that the low growth rate
(c) 1 and 2 only of India, a country with a mostly Hindu
(d) 1, 2 and 3 population, was in sharp contrast to high
Answer: A growth rates in other Asian countries,
Explanation: especially the East Asian Tigers, which were

KPIAS (23-B) CELL: 9133237733


also newly independent. This meaning of the loans from State and Union Territory
term, popularized by Robert McNamara, was Governments and other parties.
used disparagingly and has connotations that ● Capital payments consist of capital
refer to the supposed Hindu outlook of expenditure on acquisition of assets like land,
fatalism and contentedness. buildings, machinery, equipment, as also
investments in shares, etc., and loans and
78) Fiscal Responsibility and Budget management Act advances granted by the Central Government
(FRBMA) was passed to keep check on to State and Union Territory Governments,
(a) Fiscal deficit only Government companies, Corporations and
(b) Revenue deficit only other parties.
(c) Both fiscal deficit and revenue deficit ● Salaries of govt. employees are included in
(d) Neither fiscal deficit nor revenue deficit the revenue budget of the govt. of India.
Answer: C
Explanation:
The Central Government should specify four fiscal 80) Consider the following statements regarding plan
indicators- Fiscal deficit as a percentage of and non-plan expenditure:
GDP; Revenue deficit as a percentage of GDP; 1. Plan expenditure includes the assistance given by
Tax revenue as a percentage of GDP; Total the Central Government for the plans of
outstanding liabilities as percentage of GDP. States and Union Territories
2. The distinction between plan and non- plan
expenditures has been eliminated on the
79) Which of the following are included in the capital recommendation of C. Rangarajan
budget of the Government of India? Committee.
1. Expenditure on acquisition of fighter aircraft Which of the above statement is/are correct?
2. Financial assistance received from the World Bank (a) 1 only
3. Loan made to states and union territories every (b) 2 only
year (c) Both 1 and 2
4. Salaries of the government employees. (d) Neither 1 nor 2
Select the correct answer using the codes given Answer: C
below: Explanation:
(a) 1 and 2 only ● Expenditure on electricity generation,
(b) 2 and 3 only irrigation and rural developments,
(c) 1, 3 and 4 only construction of roads, bridges, canals and
(d) 1, 2 and 3 only science, technology, environment, etc. It
Answer: D includes both revenue expenditure and
Explanation: capital expenditure. Again, the assistance
The Capital Budget consists of capital receipts and given by the Central Government for the
capital payments. plans of States and Union Territories (UTs) is
● The capital receipts are loans raised by the also a part of plan expenditure. Plan
Government from the public, called market expenditure is further sub-classified into
loans, borrowings by the Government from Revenue Expenditure and Capital
the Reserve Bank and other parties through Expenditure.
sale of Treasury Bills, loans received from ● In 2011, an experts’ committee headed by C
foreign Governments and bodies, Rangarajan had proposed that the distinction
disinvestment receipts and recoveries of between Plan and non-Plan expenditure be

KPIAS (24-B) CELL: 9133237733


abolished for both the Centre and the states. ● Outcome based budgeting is a practice of
Hence, the distinction between plan and non- suggesting and listing of estimated outcomes
plan expenditures has been eliminated from of each programmes or schemes designed.
budget 2017-18 onwards. ● An interesting feature of outcome based
budgeting is that the outcomes of
programmes are measured not just in terms
81) Which of the following statements are correct of Rupees but also in terms of physical units
about the Kisan Urja Suraksha evam Utthaan like Kilowatt of energy produced or tonnes of
Mahaabhiyan (KUSUM) scheme? steel produced. Also outcomes are expressed
1. It was launched in the 11th Five Year Plan. in terms of qualitative targets and
2. It aims to incentivise farmers to run solar farm achievements to make the technique more
water pumps. comprehensive.
3. It provides for solarisation of tube-wells and lift ● Under outcome budgeting, each Ministry
irrigation projects of the Government sector. presents a preliminary Outcome Budget to
Select the correct answer using the code given below: the Finance Ministry, which is responsible for
(a) 1 and 2 only compiling them.
(b) 2 and 3 only Outcome budget is a performance measurement tool
(c) 1 and 3 only that helps in:
(d) 1, 2 and 3 ● Better service delivery
Answer: B ● Decision-making
Explanation: ● Evaluating programme performance and
● KisanUrja Suraksha evam Utthaan results
Mahaabhiyan (KUSUM) scheme was ● Communicating programme goals
announced in Budget 2018-19. It aims to ● Improving programme effectiveness
incentivise farmers to run solar farm water ● Make budgets cost effective
pumps and use barren land for generating ● Fix accountability
solar power to have extra income.
● Solarisation of tube-wells and lift irrigation
projects of the Government sector. 83) Which of the following is/are the functions of the
National Development Council of India?
(1) To prescribe guidelines for preparation of the
82) Consider the following statements about national economic plan
Outcome Budgeting: (2) To review the working of the national economic
1. Outcome budget is a performance measurement plan from time to time
tool that helps in fixing accountability. (3) To consider important questions of social and
2. In Outcome budgeting, the outcomes of economic policy affecting national
programmes are measured only in terms of development
Rupees and not in terms of physical units. Select the correct answer using the codes given
Which of the above statements is/are correct? below?
(a) 1 only (a) 1 only
(b) 2 only (b) 1 and 2 only
(c) Both 1 and 2 (c) 2 and 3 only
(d) Neither 1 nor 2 (d) 1, 2 and 3
Answer: A Answer: D
Explanation: Explanation:

KPIAS (25-B) CELL: 9133237733


The National Development Council (NDC) or the extending from 1 April 1966 to 31 March
Rashtriya Vikas Parishad 1969.
is the apex body for decision making and
deliberations on development
matters in India, presided over by the Prime Minister. 85) Which one of the following is not a
The functions of the recommendation of the Fourteenth Finance
Council are to prescribe guidelines for the Commission?
formulation of the National (a) Share of States in Central Divisible Pool is
Plan; to consider the National Plan as formulated by increased from 32 percent to 42 percent
the NITI Aayog; to (b) Area under forest cover is an important variable in
consider important questions of social and economic distribution of States’ share among States
policy affecting (c) Fiscal discipline is dropped as a variable in
national development; and to review the working of distribution of States share among States
the Plan from time to (d) Sector specific grant is recommended as in the
time and to recommend such measures as are previous Finance Commissions
necessary for achieving the Answer: C
aims and targets set out in the National Plan. Explanation:
The Fourteenth Finance Commission wants the states
to share a larger fiscal responsibility for the
84) Consider the following statements: implementation of the scheme.
(1) The Indo-china war had hampered the proper
progress and implementation of the Fourth
Five Year Plan in India. 86) Which of the following institutions was/were
(2) In the Indian Economy, the plan Holiday took asked by the Government of India to provide
place during 1966 to 1969 official estimates of black (unaccounted)
Which of the statements given above is/are correct? money held by Indians, both in India and
(a) 1 only abroad?
(b) 2 only 1. National Institute of Public Finance and Policy.
(c) Both 1 and 2 2. National Council of Applied Economic Research.
(d) Neither 1 nor 2 3. National Institute of Financial Management.
Answer: B Select the correct answer using the codes given
Explanation: below
Despite big investments during the first three Plans (a) Only 1
the living standards of the poor could not be (b) 1 and 2
raised and poverty and inequity in (c) 2 and 3
distribution of state resources remained (d) All of these
stark. Consequently, the period between Answer: D
1966 and 1969 marked the shift from a Explanation:
‘growth approach’ to a ‘distribution from The government had selected three think tanks in
growth approach’. Looking at the failures and March 2011 in order to estimate the
pitfalls, the planners suspended the quantum of black money. These three think
impending Fourth Plan, which was due in tanks are - National Institute of Financial
1966, until 1969 for a revision of objectives Management (NIFM), National Council for
and targets. This was called the ‘Plan Holiday’ Applied Economic Research (NCEAR) and

KPIAS (26-B) CELL: 9133237733


National Institute of Public Finance and Policy 89) According to the Union Budget 2023-24, consider
(NIPFP). the following statements.
1. During the Covid-19 pandemic with the PMGKY
scheme, the government supplied free food
87) Consider the following statements grains to over 80 crore persons for 28
1. Railways budget is presented together with the months.
Union budget from financial year 2017-18. 2. The theme of G20 is ‘Vasudhaiva Kutumbakam”.
2. The total receipts and expenditure of the railways Which of the above statements is/are correct?
are incorporated in the Annual Financial (a) 1 only
Statement of the government of India. (b) 2 only
3. In the budget 2023-24, ₹2.4 lakh crore capital (c) Both 1 and 2
outlay has been sanctioned for railways. (d) Neither 1 nor 2
Which of the above statements are correct? Answer: C
(a) 1 and 2 Explanation:
(b) 1 and 3 Para 5 and 6 of the Budget
(c) 2 and 3 ● During the Covid-19 pandemic, we ensured
(d) 1, 2 and 3 that no one goes to bed hungry, with a
Answer: (d) scheme to supply free food grains to over 80
Explanation: crore persons for 28 months. Continuing our
● Railways budget is presented together with commitment to ensure food and nutritional
the Union budget from financial year 2017- security, we are implementing, from 1st
18. January 2023, a scheme to supply free food
● The total receipts and expenditure of the grain to all Antyodaya and priority
railways are incorporated in the Annual households for the next one year, under PM
Financial Statement of the government of Garib Kalyan Anna Yojana (PMGKAY). The
India. entire expenditure of about ` 2 lakh crore will
● A capital outlay of Rs 2.40 lakh crore has be borne by the Central Government. G20
been provided for the Railways - the highest Presidency: Steering the global agenda
ever outlay and about 9 times the outlay through challenges
made in 2013- 14. ● In these times of global challenges, the G20
Presidency gives us a unique opportunity to
88) What is the full form of UIDF which is to be strengthen India’s role in the world economic
established as per Union Budget 2023-24? order. With the theme of ‘Vasudhaiva
(a) Urban Infrastructure Development Fund Kutumbakam’, we are steering an ambitious,
(b) Unique Identification Development Fund people-centric agenda to address global
(c) Unique Identification Infrastructure Development challenges, and to facilitate sustainable
Fund economic development.
(d) Unique Industrial Development Fund 90) According to the Union Budget 2023-24, consider
Answer: (a) the following statements.
Explanation: 1. The Vision for the Amrit Kaal includes a
2023-24 budget has sought to create urban technology-driven and knowledge-based
infrastructure in tier 2 and 3 cities via economy, with strong public finances, and a
establishment of UIDF. robust financial sector.
2. Deendayal Antyodaya Yojana National Rural
Livelihood Mission has achieved remarkable

KPIAS (27-B) CELL: 9133237733


success by mobilizing rural women into 81 2. The relationship between a nation’s GNP and NNP
lakh Self Help Groups. is similar to the relationship between its GDP
Which of the above statements is/are correct? and NDP.
(a) 1 only Which of the above statements is/are correct?
(b) 2 only (a) 1 only
(c) Both 1 and 2 (b) 2 only
(d) Neither 1 nor 2 (c) Both 1 and 2
Answer: C (d) Neither 1 nor 2
Explanation: Answer: B
Para 5 and 6 of the Budget Explanation:
● During the Covid-19 pandemic, we ensured ● Net national product (NNP) is the market
that no one goes to bed hungry, with a value of a nation’s goods and services (GNP)
scheme to supply free food grains to over 80 minus depreciation (often referred to as
crore persons for 28 months. Continuing our capital consumption).
commitment to ensure food and nutritional ● It can be found out by adding the NFIA
security, we are implementing, from 1st (National Factor Income from Abroad) to the
January 2023, a scheme to supply free food NDP. If the NFIA is positive, that is, the inflow
grain to all Antyodaya and priority of factor income from abroad is more than
households for the next one year, under PM the outflow; NNP will be more than NDP.
Garib Kalyan Anna Yojana (PMGKAY). The Conversely, if NFIA is negative, NNP will be
entire expenditure of about ` 2 lakh crore will less than NDP and it would be equal to NDP
be borne by the Central Government. G20 in case the NFIA is zero.
Presidency: Steering the global agenda ● The relationship between a nation’s GNP and
through challenges NNP is similar to the relationship between its
● In these times of global challenges, the G20 gross domestic product (GDP) and net
Presidency gives us a unique opportunity to domestic product (NDP).
strengthen India’s role in the world economic
order. With the theme of ‘Vasudhaiva 92) Consider the following with respect to the new
Kutumbakam’, we are steering an ambitious, methodology for calculation of Gross
people-centric agenda to address global Domestic Product (GDP):
challenges, and to facilitate sustainable 1. The base year for GDP calculation has been
economic development. changed to 2004-05.
2. There is a shift from factor-cost-based method to
market-cost-based method in calculation.
Which of the above statements is/are correct?
(a) 1 only
------------------------------------------- (b) 2 only
---------------------------------------------- (c) Both 1 and 2
(d) Neither 1 nor 2
Answer: B
Explanation:
91) With reference to ‘Net National Product (NNP) ‘, India has shifted to a new methodology for GDP
consider the following statements: calculation:
1. NNP is always lower than Net Domestic Product ● CSO (Central Statistical Organization) adopted
(NDP). a new base year 2011-12 instead of the

KPIAS (28-B) CELL: 9133237733


previous base year 2004-05 for estimating ● This is because the rise in GDP may be
GDP. concentrated in the hands of very few
● India’s GDP is now measured by using gross individuals or firms. For the rest, the income
value added (GVA) at market price, rather may in fact have fallen. In such a case the
than factor cost. welfare of the entire country cannot be said
to have increased.
● Disposable income, also known as disposable
93) Gross Domestic Product (GDP) is the: personal income (DPI), is the amount of
1. Monetary value of all the finished goods and money that households have available for
services produced within a country’s borders. spending and saving after deduction of
2. Measure of welfare or happiness. income taxes.
3. Measure of Disposable Personal Income.
Which of the above statements is/are correct? 94) Which of the following National Income
(a) 1 only Accounting terminologies is/are correctly
(b) 2 and 3 only matched?
(c) 1 and 3 only 1. Per capita GDP can be used to measure the
(d) 1, 2 and 3 productivity of a country’s workforce.
Answer: A 2. Personal Income serves as a measurement of the
Explanation: stability and wealth within an economy.
● Gross domestic product (GDP) is the 3. Gross Value Added gives a picture of the state of
monetary value of all the finished goods and economic activity specific to a particular
services produced within a country’s borders sector.
in a specific time period. Which of the above statements is/are correct?
● GDP includes all private and public (a) 1 Only
consumption, government outlays, (b) 2 and 3 only
investments, private inventories, paid-in (c) 1 and 3 only
construction costs and the foreign balance of (d) 1, 2 and 3
trade (exports are added, imports are Answer: D
subtracted). Put simply, GDP is a broad Explanation:
measurement of a nation’s overall economic ● Per capita GDP is a measure of the total
activity. output of a country that takes gross domestic
● GDP is commonly used as an indicator of the product (GDP) and divides it by the number
economic health of a country, as well as a of people in the country.
gauge of a country’s standard of living. Since ● The per capita GDP is especially useful when
the mode of measuring GDP is uniform from comparing one country to another, because it
country to country, GDP can be used to shows the relative performance of the
compare the productivity of various countries countries. A rise in per capita GDP signals
with a high degree of accuracy. growth in the economy and tends to reflect
● But the rise in GDP doesn’t guarantee the an increase in productivity
welfare of the masses. Gross National ● Per capita GDP can also be used to measure
Happiness (also known by the acronym: GNH) the productivity of a country’s workforce, as
is a developing philosophy as well as an it measures the total output of goods and
“index” which is used to measure the services per each member of the workforce in
collective happiness in any specific nation. a given nation.

KPIAS (29-B) CELL: 9133237733


● Personal Income serves as a measurement of Income/Factor income Method and
the stability and wealth within an economy. Expenditure Method.
● Gross value added (GVA) is the measure of
the value of goods and services produced in 96) Which of the following items are to be excluded
an area, industry or sector of an economy. from GNP measurement?
GVA gives a picture of the state of economic 1. Buying and selling of security
activity from the producers’ side or supply 2. Government transfer payments
side, the GDP gives the picture from the 3. Transfer of second hand goods
consumers’ side or demand perspective. Select the correct answer using the code given below:
(a) 3 only
(b) 2 and 3 only
95) Which of the following statements about the (c) 1 and 3 only
Product Approach of National Income (d) 1, 2 and 3
Accounting is/are correct? Answer: D
1. It computes the contribution of each producing Explanation:
unit of the economy during a given ● Gross National Product is the value of all
accounting year. goods and services produced by a country’s
2. It is a value added method of GDP estimates in residents.
which final & intermediate goods are ● GNP = GDP + Net Income from Abroad
calculated. ● The items that are to be excluded from GNP
Select the correct answer using the code given below: measurement are:
(a) 1 only ● Buying and selling of security: - Because it is
(b) 2 only merely transfer of paper from one person to
(c) Both 1 and 2 another and it does not rather add to any
(d) Neither 1 nor 2 production services.
Answer: A ● Government transfer payments: - Because it
Explanation: is one sided payment made by the
● In the Product Approach of National Income government to the household & firm without
Accounting we add up the specific value of rendering any production services. E.g.
the flows of output arising from each sector Scholarship, old age Pension etc.
of the economy. ● Transfer of used (Second hand) goods:
● It does not include intermediate goods Because it is already counted at the time of
● National income measures the income production and hence it will lead to double
generated by a country through the counting.
production activities that are carried out ● Private transfer payments: Because it is one
within a country during a specific period of sided payment made by the parents to their
time. children without rendering any productive
● A circular flow of income and expenditure services. E.g. Pocket money given by the
exists within an economy, where factor parents to their children.
income is earned from the production of
goods and services, and the income is spent
on the purchase of produced goods. Thus, 97) Which of the following statements regarding the
there are three alternative methods of estimation of National Income is/are correct?
computing national income. This includes: 1. Dadabhai Naoroji had prepared the first estimates
Product/ Value Added Method, of National Income.

KPIAS (30-B) CELL: 9133237733


2. Dr. VKRV Rao used scientific procedure in ● National Income is the total value of all final
estimating the National Income. goods and services produced by the country
Select the correct answer using the code given below: in a certain year.A total of national income
(a) 1 Only measures the flow of goods and services in an
(b) 2 Only economy. The growth of National Income
(c) Both 1 and 2 helps to know the progress of the country. It
(d) Neither 1 nor 2 includes payments made to all resources in
Answer: C the form of wages, interest, rent and profits.
Explanation: National income is also defined as “the net
● Dadabhai Naoroji, fondly called the Grand output of commodities and services flowing
Old Man of India, was the pioneer in this during the year from the country’s productive
field. He prepared the first estimates of system in the hands of the ultimate
National income in 1876. He estimated the consumers.
national income by first estimating the value ● National Wealth not National Income
of agricultural production and then adding a measures the stock of commodities held by
certain percentage as non-agricultural the nationals of a country at a point of time.
production. However, such method can only ● As contrasted with national wealth which
been called as a non-scientific method. measures the stock of commodities held by
● The first person to adopt a scientific the nationals of a country at a point of time,
procedure in estimating the national income national income measures the productive
was Dr. VKRV Rao in 1931. He divided the power of an economy in a given period to
Indian Economy into two parts: Agricultural turn out goods and services for final
Sector which included agriculture, forests, consumption.
fishing and hunting and Corporate Sector
which included industries, construction,
business, transport and public services. 99) Which of the following entities are included while
calculating Gross National Income?
1. Profit
98) Consider the following statements with respect to 2. Wages
National Income: 3. Interest
1. National income measures the flow of goods and Select the correct answer using the code given below:
services in an economy. (a) 1 and 2 only
2. National income measures the productive power of (b) 1 and 3 only
an economy to convert (c) 2 and 3 only
commodities for final consumption in a given period. (d) 1, 2 and 3
3. National income measures the stock of Answer: D
commodities held by the nationals of a Explanation:
country at a given point of time. Gross National Income
Which of the above statements is/are correct? ● The ‘income method’ calculation of Gross
(a) 1 Only National Income is ‘Profit +Wage +Interest
(b) 1 and 2 Only +Rent’.
(c) 1 and 3 Only ● It is the sum of a nation’s gross domestic
(d) 1, 2 and 3 product and the net income it receives from
Answer: B overseas.
Explanation:

KPIAS (31-B) CELL: 9133237733


● Gross National Income (GNI) is a measure of a 2. Per Capita Income of any nation is National Income
country’s income. While GDP only counts divided by its workforce.
income received from domestic sources, GNI Which of the above statements is/are correct?
includes Net Income Received from abroad. (a) 1 Only
(b) 2 Only
(c) Both 1 and 2
100) Which of the following is/are major indicators of (d) Neither 1 nor 2
the State of an Economy? Answer: A
1. Rate of GDP growth Explanation:
2. Rate of inflation ● India’s GDP at PPP is more than 3 times that
3. Change in Net demand and time liabilities GDP at market.
Select the correct answer using the code given below: ● Per Capita Income is National Income divided
(a) 1 Only by the entire population of the nation.
(b) 2 Only
(c) 1 and 2 only
(d) 1, 2 and 3 102) Which of the following transactions is/are not
Answer: C included in calculating the GDP of a nation?
Explanation: 1. Money received from selling old car
● Rate of GDP growth is a major indication of 2. Social Security benefit transfer
the state of the economy of a country. Select the correct answer using the code given below.
Economic growth is the increase in the (a) 1 Only
market value of the goods and services (b) 2 Only
produced by an economy over time. It is (c) Both 1 and 2
conventionally measured as the percent rate (d) Neither 1 nor 2
of increase in gross domestic product. Gross Answer: C
domestic product (GDP) is the market value Explanation:
of all officially recognized final goods and ● Gross Domestic Product is simply the value of
services produced within a country in a year, all final goods and services that a nation
or other given period of time. produces during a given period-usually a
● The Macroeconomic Vulnerability Index, as year, minus imports.
mentioned in the Mid-Year Economic Analysis ● It also excludes Financial Transactions,
2014-2015, released by the Ministry of Secondhand Goods and Transfer Payments.
Finance recently, adds together the rate of Many transactions occur that have nothing to
inflation, current account deficit and fiscal do with final goods and services
deficit of a country. The Index value can be produced/GNP.
compared across countries for different time Financial Transactions excluded:
periods to gauge their relative vulnerability ● Buying and Selling Securities: The value of
and state of the economy. broker’s services is included in GDP because
they perform a service.
● Government Transfer Payments: Transfer
payments are payments for which no
productive services are concurrently provided
101) Consider the following statements: in exchange. Social Security Transfers etc.
1. India’s GDP at Purchasing Power Parity is higher
than its GDP at Market Exchange Rate.

KPIAS (32-B) CELL: 9133237733


● Private Transfer Payments: This is a private (d) Neither 1 nor 2
transfer of funds from one person to another Answer: C
and these are not included in GDP. Explanation:
● In economics, the Lorenz curve is a graphical
representation of the distribution of income
or of wealth.
103) Which of the following measures can reduce the ● It was developed by Max O. Lorenz in 1905
Debt-GDP ratio of an economy? for representing inequality of the wealth
1. Increase in Economic growth. distribution.
2. Decrease in Government Spending. ● A straight line on it represents complete
3. Increase in Tax Income Revenue. equality of income.
Select the correct answer using the code given below: ● With greater curvature in it, the inequality of
(a) 1 only income rises proportionally, this inequality is
(b) 1 and 2 only measured by the ‘Gini Coefficient’
(c) 2 and 3 only
(d) 1, 2 and 3
Answer: D 105) Which of the following statements is correct?
Explanation: (a) GDP at factor cost = Net Value Addition +
The debt- to-GDP ratio itself is an equation with a Depreciation
country’s gross debt in the numerator and its (b) GDP at factor cost = Net Value Addition -
gross domestic product (GDP) in the Depreciation
denominator. (c) GDP at factor cost = Net price increase + indirect
In general, the debt-to-GDP ratio is used to tax
determine the health of an economy. The (d) GDP at factor cost = Net price increase + direct tax
debt-to-GDP ratio is designed to help Ans: a
investors determine if a country has too Explanation: GDP at factor cost = Net Value Addition
much debt. + Depreciation is correct. GDP at a factor cost
● Debt/GDP ratios can be reduced by applying is the sum of net value addition by all
the following measures: producers within the country.
● Increase in Economic growth
● Decrease in Government Spending. 106) Which statement is correct for nominal GDP?
● Increase in Tax Income. 1. Nominal GDP is calculated based on current
prices.
2. Nominal GDP is calculated based on the base
104) Consider the following statements about the prices.
‘Lorenz Curve’: 3. Data on Nominal GDP shows an accurate
1. A straight line on it represents complete equality of picture of the economy as compared to real
income. GDP.
2. With greater curvature in it, the inequality of Options:
income rises proportionally, this inequality is (a) Only ii, iii
measured by the ‘Gini Coefficient’. (b) only ii
Which of the above statements is/are correct? (c) only i
(a) 1 only (d) i, iii
(b) 2 only Ans: c
(c) Both 1 and 2

KPIAS (33-B) CELL: 9133237733


Explanation: Nominal GDP is calculated on the basis Explanation:
of current prices. While real GDP is calculated Per capita income is a measure of the average income
on the base year prices and its data are more earned per person in a given area (usually in
reliable or accurate as compared to Nominal a country) in a particular year. When this
GDP. figure is adjusted for inflation, the real per
capita income is obtained, which gives the
107) The value of which work is added in the best measure of an increase in a country’s
calculation of GDP? economic efficiency.
(a) Housewives' works
(b) A teacher teaching his own child
(c) The value of resale of old shares
(d) Construction of new house by an artisan 110) What does the term 'Green shoots' represent in
Ans: d an Economy?
Explanation: The value of the construction of a new (a) signs of growth of the agriculture sector in a
house is added in the calculation of the GDP growing economy
because it is a new production work in the (b) signs of economic recovery during an economic
economy. downturn.
(c) signs of growth of the agriculture sector in a
declining economy
(d) signs of economic decline in a developed
108) Which among the following is a most suitable economy.
example of double counting in national Answer: b
income? Explanation:
(a) Wages of bus and train drivers. Green shoots is a term used colloquially and
(b) Cotton output and cotton cloth output propagandistically to indicate signs of
(c) Electricity output and water output. economic recovery during an economic
(d) Tax receipts and earnings of inland revenue downturn. It was first used in this sense by
officials. Norman Lamont, the then Chancellor of the
Answer: b ~xche~uer of the United Kingdom, during the
Explanation: 1991 recession.
While estimating the national income, the problem of
double counting occurs when the value of
some goods and services are counted more 111) GDP deflator is used to:
than once. Cotton output and cotton cloth (a) measure the relative reduction in GDP growth rate
output both the raw material and the final of a country.
result are counted. (b) measure the inflation in a country.
(c) compare the GDP of a country vis a vis other
109) Which among the following is considered to be countries of the world.
the best measure of an increase in a country's (d) estimate the purchasing power of the citizens of a
economic efficiency? country.
(a) Increase in annual private investment. Answer: b
(b) Increase in real national income. Explanation:
(c) Increase in real per capita income. GDP deflator is an economic metric that accounts for
(d) Increase in net annual investment. inflation by converting output measured at
Answer: c current prices into constant-collar GDP. The

KPIAS (34-B) CELL: 9133237733


GDP deflator shows how much a change in Explanation:
the base year's GDP relies upon changes in National product at factor cost is equal to net
the price level. domestic product at factor cost + Net factor
Income from Abroad.

112) National income ignores which one of the


following? 115) Which of the following is a part of the Gross
(a) sales of a form National Product (GNP)?
(b) salary of employees A. Exports
(c) exports of the IT sector B. Money earned by a resident abroad
(d) sale of land C. Imports
Answer: d D. All of the above
Explanation: Answer: d
National Income ignores sale of land. Explanation:
In calculation, GNP adds government expenditure,
personal consumption expenditure, private
113) The value of all final goods and services domestic investments, net exports, and
produced by the normal residents of a income earned by nationals overseas, and
country and their property, whether eliminates the income of foreign residents
operating within the domestic territory of the within the domestic economy.
country or outside in a year is termed as
(a) Gross National Income 116) The net value of Gross Domestic Product after
(b) Net National Income deducting depreciation from it is _______.
(c) Gross Domestic Product A. Net Domestic Product
(d) Net Domestic Product B. Net National Product
Answer: a C. Disposable Income
Explanation: D. Gross National Product
The sum of a nation's gross domestic product (GDP) Answer: a
plus net income received from overseas. Explanation:
Gross national income (GNI) is defined as the The net value of GDP after deducting depreciation
sum of value added by all producers who are from GDP is Net domestic product.
residents in a nation, plus any product taxes
(minus subsidies) not included in output, plus
income received from abroad such as 117) Which of the following is not a part of National
employee compensation and property Income?
income. A. Undistributed profit
114) National product at factor cost is equal to B. The payments made by the household to the
(a) Domestic product + Net factor income from firm for purchasing goods and services
abroad C. Income from government expenditure
(b) National product at market prices - indirect taxes D. Interest on the unproductive national debt
+ subsidies Answer: d
(c) Gross domestic-product - depreciation Explanation:
(d) National product at market prices + Indirect taxes National income doesn't include interest on the
+ subsidies unproductive national debt. National income
Answer: a defines the value of goods and services

KPIAS (35-B) CELL: 9133237733


produced by any country in the period of a A. Includes consumer spending on durable and
financial year. non-durable goods as well as services
B. Includes consumer spending on durable and
non-durable goods but excludes services
118) Which of the following is a limitation of the C. Includes purchases by business firms
Gross Domestic Product? D. None of the above
A. It fails to indicate the sustainability of a Answer: a
nation’s growth Explanation:
B. It doesn’t consider the impact of economic While using expenditure to calculate Gross Domestic
activities on human health and the Product, consumption includes consumer
environment spending on durable and non-durable goods
C. It does not include non-market transactions as well as services.
D. All of the above
Answer: d 121) Consider the following statements regarding
Explanation: contribution of primary sector
GDP does not include the profits of domestic i) According to census 2011 About 54.6 percent
companies operating abroad. Gross national of the population is engaged in agriculture
product (GNP) is the total value of goods and and allied activities.
services produced and income received by ii) Agriculture and allied activities contributed
the domestic residents in a financial year. It 22.8 percent to the country’s Gross Value
means that any product made by the Added for the year 2019-20.
country's residents must be included in the Which of the statements given above are correct?
national income, whether inside or outside A) Only i
the country. However, gross domestic B) Only ii
product (GDP) does not include the profits of C) Both
domestic companies operating abroad, which D) None
is one of the limitations of GDP. Ans: A
AGRICULTURE plays a vital role in India’s economy.
119) Which of the following is added in calculating About 54.6 per cent of the population is
Gross Domestic Product? engaged in agriculture and allied activities
A. A teacher teaching their child (Census 2011) and it contributed 17.8 per
B. The value of resale of old shares cent to the country’s Gross Value Added for
C. An artisan constructing a new house the year 2019-20 (at current prices). Given
D. None of the above the importance of agriculture sector,
Answer: c Government of India takes several steps for
Explanation: its sustainable development.
The value of the construction of a new house is 122. Which among the following States have
added in the calculation of the GDP because derived maximum benefits from green
it is a new production work in the economy. revolution?
A) Bihar, West Bengal and Assam
B) Rajasthan, Gujarat and Maharashtra
120) While using expenditure to calculate Gross C) Punjab, Haryana & Western U.P.
Domestic Product, consumption D) Tamil Nadu, Andhra Pradesh and
___________. Maharashtra
Ans: C

KPIAS (36-B) CELL: 9133237733


Green revolution in India started with Kharif crop in farmers for all Kharif crops and 1.5 per cent
1966. The States like Punjab, Haryana, Uttar for all Rabi crops. In case of annual
Pradesh (Western part), etc. having good commercial and horticultural crops, the
irrigation and other infrastructure facilities maximum premium to be paid by farmers is
were able to derive the maximum benefits of up to 5 per cent.
the green revolution.
125. Under the automatic route approach, the
123. India is the world’s largest producer of? government of India permitted how much
i) Milk percent of FDI in food product marketing and
ii) Pulses food product e-commerce?
iii) Spices A) 49 percent
iv) Eggs B) 51 percent
Select the answer using the codes given below C) 75 percent
A) Only I, ii and iii D) 100 percent
B) Only I and iii Ans: D
C) Only I, iii and iv Agriculture is the primary source of livelihood for
D) All the above more than 50 percent of India’s population.
Ans: A Under the automatic route approach, the
India is the world’s largest producer of milk, pulses government of India permitted 100 percent
and spices, and has the worlds largest cattle of FDI in food product marketing and food
herd as well as the largest area under wheat, product e-commerce.
rice and cotton.
126. Which among the following is the nodal agency
124. Consider the following statements regarding the to coordinates and promotes agricultural
Pradhan Mantri Fasal Bima Yojana research and education in the country
i) A uniform maximum premium of only 2 through its autonomous bodies to achieve
percent of the sum insured is paid by farmers the ambitious goal of doubling farm income
for all Kharif crops for both rabi and kharif by 2022?
crops under this scheme. A) NITI Aayog
ii) Pradhan Mantri Fasal Bima Yojana was B) Department of Agriculture and Farmers
launched in 2014. Welfare
Which of the statements given above are correct? C) Department of Agricultural Research and
A) Only i Education
B) Only ii D) Department of organic farming
C) Both Ans: C
D) None The present government’s target is to achieve the
Ans: D ambitious goal of doubling farm income by
Pradhan Mantri Fasal Bima Yojana (PMFBY), which is 2022. Towards this target, the government
being implemented in various states/union has increased investment in agricultural
territories of the country from Kharif 2016. infrastructure such as irrigation facilities,
The scheme is being implemented through 18 warehousing and cold storage. Towards these
General Insurance Companies including all basic goals, DARE coordinates and promotes
the 5 Government Sector Companies. Under agricultural research and education in the
PMFBY, a uniform maximum premium of only country through its autonomous bodies, viz.
2 per cent of the sum insured is paid by Indian Council of Agricultural Research

KPIAS (37-B) CELL: 9133237733


(ICAR), Central Agricultural University (CAU), output of national agricultural research
Imphal, Dr. Rajendra Prasad Central system.
Agricultural University (DRPCAU), Pusa, Bihar,
and Rani Lakshmi Bai Central Agricultural 129. Consider the following statements regarding the
University (RLBCAU), Jhansi, UP. MSP
i) The Government of India announced to fix
127. Consider the following factors the MSP for crops 50 percent above their
i) Cost of Production production cost since 2018–19.
ii) Demand and supply, ii) The new MSP is based on the formula that
iii) Price trend in the domestic and international over the Cost A2+Fl.
markets iii) Cost A2 includes imputed cost of unpaid
Which of the above were the important factors in the family labour.
determination of MSP of mandated crops? Which of the statements given above are incorrect?
A) Only I and ii A) Only iii
B) Only iii B) Only ii and iii
C) Only i C) Only I and ii
D) All the above D) All the above
Ans: D Ans: A
Cost of Production (CoP) is one of the important The Government announced to fix the MSP for crops
factors in the determination of MSP of 50 per cent above their production cost since
mandated crops. Besides cost, the 2018–19. The new MSP is fixed by the CACP
Commission considers other important over the Cost A2+Fl (where, Cost A2 includes
factors such as demand and supply, price all paid out costs borne by the farmer on
trend in the domestic and international seeds, fertilisers, pesticides, hired labour,
markets, intercrop price parity, terms of leased-in land, fuel, irrigation, etc. and the Fl
trade between agricultural and non- is imputed cost of unpaid family labour).
agricultural sectors and the likely impact of
MSP on consumers, in addition to ensuring 130. Consider the following statements regarding the
rational utilisation of natural resources like Price Stabilisation Fund
land and water. i) The Government of India launched the Price
Stabilisation Fund in March 2019.
128. National Agriculture Technology Project (NATP) ii) The main objective of the scheme is to
is funded in India by which among the support market interventions for price
following? control of perishable Agri horticultural
A) Food and Agriculture Organization commodities.
B) World Bank Which of the statements given above are correct?
C) Asian Development Bank A) Only i
D) International Monetary Fund B) Only ii
Ans: B C) Both
National Agriculture Technology Project (NATP) was D) None
launched by the Indian Council of Agricultural Ans: B
Research (ICAR) and Union Ministry of The Government of India, by late March 2015,
Agriculture in 1998, with the support of the launched the Price Stabilisation Fund (PSF) as
World Bank to strengthen and complement a Central Sector Scheme to support market
the existing resources and to augment the interventions for price control of perishable

KPIAS (38-B) CELL: 9133237733


Agri horticultural commodities. The cost to be feed and fodder development. The scheme is
borne between the centre and the states in implemented with the following three sub-
equal ratio (in case of the North Eastern- missions. Sub-Mission on Feed and Fodder
states, the respective share will be 75:25). Development: This sub-mission aims towards
The scheme will commence with only two strengthening the fodder seed chain to
crops, viz., onion and potato. improve availability of certified fodder seed
required for fodder production and
131. Consider the following characteristics encouraging entrepreneurs for establishment
i) high yielding variety (HYV) seeds, of fodder block/hay, bailing /silage making
ii) chemical fertilisers, units through incentivisation.
iii) insecticides and pesticides
Which of the above are the main characteristics of 133. Consider the following challenges
commercial farming? i) fragmented landholdings,
A) Only I and ii ii) sub-optimal farm mechanisation,
B) Only i iii) low productivity,
C) Only ii and iii iv) institutional unemployment,
D) All the above v) lowering input costs
Ans: D Which of the above were certain challenges
Commercial Farming: The main characteristic of this specifically associated with the Indian
type of farming is the use of higher doses of agriculture?
modern inputs, e.g., high yielding variety A) Only I, iii, iv and v
(HYV) seeds, chemical fertilisers, insecticides B) Only I, ii and iii
and pesticides in order to obtain higher C) Only iv and v
productivity. The degree of D) All the above
commercialisation of agriculture varies from Ans: B
one region to another. For example, rice is a While Indian agriculture has performed well, the
commercial crop in Haryana and Punjab, but sector needs re-orientation in the backdrop
in Odisha, it is a subsistence crop. of certain challenges like adverse impacts of
climate change, fragmented landholdings,
sub-optimal farm mechanisation, low
132. Sub-Mission on Feed and Fodder Development productivity, disguised unemployment, rising
sub-mission aimed towards strengthening the input costs, etc.
fodder seed chain to improve availability of
certified fodder seed required for fodder 134. Assertion (A): The year 2022 experienced a
production is a sub-mission of? decline in the sown area for paddy cultivation
A) Rastriya Gokul Mission too in the Kharif season due to delayed
B) National Livestock Mission monsoons and deficient rainfall.
C) Bharat Jaivik Yojana Reason (R): As per First Advance Estimates 2022-23,
D) Atal Nakul Yojana (AKY) the paddy area was about 3.8 lakh hectares
Ans: B more than the sown area during 2021-22.
National Livestock Mission (NLM) scheme has been Select the answer using the codes given below
restructured from 2021-22 to 2025-26, The A) Both A and R are true, and R is proper
focus of the scheme is on entrepreneurship explanation of A
development and breed improvement in B) Both A and R are true, and R is not proper
poultry, sheep, goat and piggery including explanation of A

KPIAS (39-B) CELL: 9133237733


C) Only A is true D) None
D) Only R is true Ans: C
Ans: C Farm mechanisation helps increase productivity
The year 2022 witnessed an early heat wave during through timely and efficient use of other
the wheat-harvesting season, adversely inputs and natural resources while at the
affecting its production. The year same time reducing the cost of cultivation
experienced a decline in the sown area for and the drudgery associated with various
paddy cultivation too in the Kharif season due farm operations. Under the Sub Mission on
to delayed monsoons and deficient rainfall. Agricultural Mechanisation (SMAM), State
As per First Advance Estimates 2022-23 Governments are being assisted in training
(kharif only) the paddy area was about 3.8 and demonstrating agricultural machinery
lakh hectares less than the sown area of and helping farmers procure various farm
411.2 lakh hectare during 2021-22. machinery and equipment besides setting up
Custom Hiring Centres (CHC).
135. The Government of India extending the Kisan
Credit Card Scheme facility to fisheries and 137. Consider the following statements regarding the
animal husbandry farmers in? organic farming
A) 2016-17 i) India has the highest number of organic
B) 2020-21 farmers in the world.
C) 2021-22 ii) About 59.1 lakh ha of area was brought under
D) 2018-19 organic farming in India by 2021-22.
Ans: D Which of the statements given above are incorrect?
Accordingly, the Kisan Credit Card Scheme (KCC) was A) Only i
introduced in 1998 for farmers to empower B) Only ii
them to purchase agricultural products and C) Both
services on credit at any time. As of 30 D) None
December, 2022, banks issued Kisan Credit Ans: D
Cards (KCC) to 3.89 crore eligible farmers Organic and natural farming provides chemical
with a KCC limit of ₹4,51,672 crore. With the fertiliser and pesticide-free food grains and
Government of India extending the KCC other crops, improves soil health and reduces
facility to fisheries and animal husbandry environmental pollution. India has 44.3 lakh
farmers in 2018-19. organic farmers, the highest in the world, and
about 59.1 lakh ha area was brought under
136. Consider the following statements regarding the organic farming by 2021-22. Sikkim
Farm mechanisation voluntarily adopted going organic state.
i) Farm mechanisation helps increase
productivity through timely and efficient use
of other inputs and natural resources. 138. PKVY Scheme is being implemented in a cluster
ii) Farm mechanisation helps in reducing the mode. Financial assistance of how much per
cost of cultivation and the drudgery ha is provided to the farmer under the
associated with various farm operations. scheme?
Which of the statements given above are correct? A) ₹50,000
A) Only i B) ₹1,00,000
B) Only ii C) ₹75,000
C) Both D) ₹30,000

KPIAS (40-B) CELL: 9133237733


Ans: A 140. Assertion (A): Promotion of natural farming in
Paramparagat Krishi Vikas Yojana (PKVY) and Mission India began in 2021-22.
Organic Value Chain Development for North Reason (R): Bhartiya Prakratik Krishi Paddhati (BPKP)
Eastern Region (MOVCDNER) since 2015 is a sub-scheme of Paramparagat Krishi Vikas
through cluster/ Farmer Producer Yojana was launched to assist farmers in
Organisations (FPOs) formation. PKVY adopting traditional indigenous practices for
Scheme is being implemented in a cluster encouraging all forms of ecological farming.
mode (with min. 20 ha size). Financial Select the answer using the codes given below
assistance of ₹50,000 per ha for three years is A) Both A and R are true, and R is proper
provided to the farmer, out of which ₹31,000 explanation of A
is given as incentives for organic inputs B) Both A and R are true, and R is not proper
provided directly through Direct Benefit explanation of A
Transfer (DBT). C) Only A is true
D) Only R is true
139. Consider the following statements regarding the Ans: D
agriculture marketing Promotion of natural farming began in 2019-20, when
i) The scope of the Essential Commodities Act, Bhartiya Prakratik Krishi Paddhati (BPKP), a
1955 is much broader than the APMC Act. sub-scheme of PKVY, was launched to assist
ii) Essential Commodities Act, 1955 empowers farmers in adopting traditional indigenous
only central government to control practices for encouraging all forms of
production, supply and distribution of certain ecological farming, including Zero-Budget
commodities. Natural Farming (ZBNF).
Which of the statements given above are correct?
A) Only i
B) Only ii 141. Consider the following statements regarding the
C) Both i) Agriculture Infrastructure Fund is a financing
D) None facility operational from the year 2020-21 to
Ans: A 2030-40.
The scope of the Essential Commodities Act, 1955 (EC ii) The main objective of the Agriculture
Act) is much broader than the APMC Act. It Infrastructure Fund is to create post-harvest
empowers the central and state governments management infrastructure and community
concurrently to control production, supply farm assets.
and distribution of certain commodities, Which of the statements given above are incorrect?
including pricing, stock-holding and the A) Only i
period for which the stocks can be kept and B) Only ii
to impose duties. The APMC Act on the other C) Both
hand, controls only the first sale of the D) None
agricultural produce. Apart from food-stuffs Ans: A
which are covered under the APMC Act, the Agriculture Infrastructure Fund (AIF): AIF is a
commodities covered under the EC Act financing facility operational from the year
generally are: drugs, fertilisers, textiles and 2020-21 to 2032-33 for the creation of post-
coal. harvest management infrastructure and
community farm assets, with benefits
including 3 per cent interest subvention and
credit guarantee support. Under this, a

KPIAS (41-B) CELL: 9133237733


provision of ₹1 lakh crore for 2020-21 to which fall into the blue and green boxes.
2025-26 has been made, and interest These include government policies of
subvention and credit guarantee assistance minimum support prices (as MSP in India) for
will be given until 2032-33. AIF scheme has agricultural products or any help directly
the facility of convergence with any other related to production quantities (as power,
scheme of the State or Central Government fertilizers, pesticides, irrigation, etc).
and can prove to be a milestone in
investment in the agriculture sector. 144. Consider the following crops
i) fruits,
142. Consider the following statements regarding the ii) root
Pradhan Mantri Fasal Bima Yojana iii) tuber crops,
i) Compulsory participation for all farmers. iv) spices,
ii) Selection of Insurance Companies by the v) plantation
States for 3 years in a go. Mission for Integrated Development of Horticulture
iii) Two-step process of crop yield estimation. aimed to promote horticulture covering crops
iv) Use of smart sampling technique through of which among the above?
satellite data for crop cutting experiments. A) Only I, ii, iii and iv
The revamped Pradhan Mantri Fasal Bima Yojana has B) Only I, iii, iv and v
many features which include? C) Only I, iii and v
A) Only I, ii and iii D) All the above
B) Only ii, iii and iv Ans: D
C) Only iii and iv Mission for Integrated Development of Horticulture
D) All the above (MIDH): Several expert groups have identified
Ans: B horticulture as a high-growth area and a
The revamped Pradhan Mantri Fasal Bima Yojana source of buoyant income and improved
w.e.f Kharif 2020 has many features, which resilience for farmers. The scheme to
include voluntary participation for all promote horticulture covering fruits,
farmers, selection of Insurance Companies by vegetables, root and tuber crops, spices,
the States for 3 years in a go, a two-step flowers, plantation crops etc., was introduced
process of crop yield estimation, use of smart in 2014-15.
sampling technique through satellite data for
crop cutting experiments, etc 145. Consider the following statements regarding
New Urea Policy
143. Minimum Support Prices policies of the i) The New Urea Policy was notified in 2014.
Government of India for agricultural products ii) The main objective of the policy is to
fall into which among the following maximising production of food grains and
categories of WTO subsidies? pulses.
A) Blue Box Which of the statements given above are incorrect?
B) Amber Box A) Only i
C) Green Box B) Only ii
D) Social and Development Box C) Both
Ans: B D) None
All subsidies which are supposed to distort Ans: C
production and trade fall into the amber box, The New Urea Policy-2015 was notified with the
i.e., all agricultural subsidies except those objectives of maximising indigenous urea

KPIAS (42-B) CELL: 9133237733


production; promoting energy efficiency in A) Only i
urea production; and rationalising subsidy B) Only ii
burden on the government. It is expected C) Both
that the domestic urea sector would become D) None
globally competitive in terms of energy Ans: B
efficiency over a period of three years. Climate-Smart Farming Practices: This is slowly
gaining acceptance with farmers using clean
146. Consider the following statements regarding the energy sources like solar for irrigation. The
e-NAM scheme farmers have been incentivised to transfer
i) The Government of India launched the electricity generated through solar to the
National Agriculture Market (e-NAM) Scheme local grid. Crop yield prediction models using
in March 2019. artificial intelligence and drones for
ii) The main aim of the e-NAM scheme is to monitoring soil and crop health have been
create an online transparent, competitive initiated. Smart farming also enables crop
bidding system to ensure farmers get diversification, which will help farmers
remunerative prices for their produce. reduce their dependence on monsoons for
Which of the statements given above are incorrect? water. There are over 1,000 aggrotech start-
A) Only i ups in India. These assist farmers in
B) Only ii improving farming techniques.
C) Both
D) None 148. Consider the following statements regarding the
Ans: A primary sector
National Agriculture Market (e-NAM) Scheme: The i) Animal Husbandry infrastructure
Government of India launched the National Development Fund was launched as part of
Agriculture Market (e-NAM) Scheme in 2016 Atma Nirbhar Bharat Abhiyan stimulus
to create an online transparent, competitive package.
bidding system to ensure farmers get ii) Animal Husbandry infrastructure
remunerative prices for their produce. Under Development Fund facilitates for
the e-NAM Scheme, the Government incentivisation of investments in
provides free software and assistance of ₹75 establishment of infrastructure for dairy and
lakh per APMC mandi for related hardware, meat processing and value addition
including quality assaying equipment and the infrastructure.
creation of infrastructure like cleaning, Which of the statements given above are correct?
grading, sorting, packaging, compost unit, A) Only i
etc. B) Only ii
C) Both
147. Consider the following statements regarding the D) None
Climate-Smart Farming Practices Ans: C
i) The farmers have been incentivised to Climate-Smart Farming Practices: This is slowly
transfer electricity generated through solar to gaining acceptance with farmers using clean
the local grid under the Climate-Smart energy sources like solar for irrigation. The
Farming Practices. farmers have been incentivised to transfer
ii) Climate Smart farming disables crop electricity generated through solar to the
diversification. local grid. Crop yield prediction models using
Which of the statements given above are incorrect? artificial intelligence and drones for

KPIAS (43-B) CELL: 9133237733


monitoring soil and crop health have been
initiated. Smart farming also enables crop
diversification, which will help farmers
reduce their dependence on monsoons for
water. There are over 1,000 Agri-tech start-
ups in India. These assist farmers in
improving farming techniques.

149. Green Revolution in India started from which of


the following region?
A) Pantnagar
B) Amritsar
C) Varanasi
D) Chennai
Ans. A
The beginning of green revolution in India is
considered from the year 1966. University of
Agriculture, Pantnagar has been an important
contributor in bringing green revolution in
India. This university is also called as the birth
place of green revolution in India.

150. FSSAI is promoting food fortification by notifying


standards for fortification of which of the
following key staple foods?
i) Milk,
ii) Oil
iii) wheat,
iv) rice
v) salt
Select the answer using the codes given below
A) Only I, iii and iv
B) Only iii, iv and v
C) Only I, ii, iv and v
D) All the above
Ans: D
To address malnutrition, FSSAI is promoting food
fortification by notifying standards for
fortification of five key staples viz., Milk, Oil
(with Vit A & D), wheat, rice (with iron, folic
acid and Vit B12 with 6 other nutrients as
voluntary addition) and salt (double fortified
with iron and iodine). +F logo has been
introduced for identification of fortified food.

KPIAS (44-B) CELL: 9133237733

You might also like